Top Banner
Print Preview Chapter 11 Neurologic Disorders Kerri Kissel Neil Porter I. Approach to the Patient with a Neurologic Complaint A. Patient history The patient history is the cornerstone of neurologic assessment. 1. Key questions. Questions that may help direct the patient interview include: o a. Was the onset of symptoms gradual or sudden? o b. Are the symptoms static, intermittent, or progressive? o c. Has the problem remained limited in scope, or have new features been introduced over time? o d. What concurrent problems does the patient have, and what medications or drugs are being used? o e. Is there a family history of the disorder or predisposing conditions? o f. What habits and toxin exposures might the patient have? 2. Review of symptoms. Depending on the clinical complaint, a patient should be asked whether there is any history of: o a. Headache or trauma to the head, neck, or spine o b. Loss of consciousness, convulsive activity, mood alterations, confusion, or memory disturbances o c. Impaired or double vision, facial numbness or weakness, impaired hearing or swallowing, or abnormal speech o d. Arm or leg weakness or heaviness, slowness of movement, altered limb sensation, discomfort or tingling in the extremities o e. Clumsiness, falling, or dizziness o f. Bowel or bladder disturbances or sexual dysfunction
143
Welcome message from author
This document is posted to help you gain knowledge. Please leave a comment to let me know what you think about it! Share it to your friends and learn new things together.
Transcript
Page 1: Neurology

Print PreviewChapter 11Neurologic DisordersKerri KisselNeil PorterI. Approach to the Patient with a Neurologic ComplaintA. Patient historyThe patient history is the cornerstone of neurologic assessment.

1. Key questions. Questions that may help direct the patient interview include:o a. Was the onset of symptoms gradual or sudden?o b. Are the symptoms static, intermittent, or progressive?o c. Has the problem remained limited in scope, or have new features

been introduced over time?o d. What concurrent problems does the patient have, and what

medications or drugs are being used?o e. Is there a family history of the disorder or predisposing conditions?o f. What habits and toxin exposures might the patient have?

2. Review of symptoms. Depending on the clinical complaint, a patient should be asked whether there is any history of:

o a. Headache or trauma to the head, neck, or spineo b. Loss of consciousness, convulsive activity, mood alterations,

confusion, or memory disturbanceso c. Impaired or double vision, facial numbness or weakness, impaired

hearing or swallowing, or abnormal speecho d. Arm or leg weakness or heaviness, slowness of movement, altered

limb sensation, discomfort or tingling in the extremitieso e. Clumsiness, falling, or dizzinesso f. Bowel or bladder disturbances or sexual dysfunction

B. Neurologic examinationFrom the patient history, the physician can generate a series of diagnostic hypotheses that can be tested with a focused neurologic examination. Anatomic localization of the pathology within the nervous system is essential to this process (Figure 11-1).

1. Mental status . If the patient's mental status is abnormal, the history and those components of the physical examination that depend on patient cooperation must be approached within the proper context. For example, if the patient is confused, the sensory examination may be unreliable.

o a. The patient's level of arousal, orientation, short- and long-term memory, affect (i.e., mood), concentration and attention, fund of knowledge, insight, judgment, and constructional ability should be assessed.

o b. Linguistic abilities are evaluated by examining comprehension, repetition, fluency, naming, reading, and writing.

o c. The integrity of other cortical functions (e.g., graphesthesia, stereognosis, two-point discrimination, right–left orientation, and neglect) should be examined if parietal lobe dysfunction is suspected.

2. Cranial nerves . Examination of cranial nerves II–XII is necessary (Table 11-1).

Page 2: Neurology

o a. In particular, visual acuity and fields should be checked; the optic nerve should be examined; and abnormalities of ocular motility, including nystagmus and dysmetria, should be documented (Table 11-2).

o b. Abnormalities of facial sensation (including the corneal reflex) and movement also should be investigated.

View Figure

FIGURE 11-1 Summary of some of the outstanding neurologic signs and symptoms that occur with focal destructive lesions in the right or left cerebral hemisphere as detected on neurologic examination. (A) Lateral view of the left cerebral hemisphere. (B) Lateral view of the right cerebral hemisphere. (Reprinted from NMS Neuroanatomy. Malvern, PA: Harwal Publishing, 1988:314 ).

TABLE 11-1 Twelve Cranial Nerves

CNNerveFunctionIOlfactorySmellIIOpticVisionIIIOculomotorEye movementsIVTrochlearEye depression (when adducted)VTrigeminalFacial sensationVIAbducensEye abductionVIIFacialFacial movementVIII

VestibulocochlearHearing

IXGlossopharyngealPalatal sensationXVagusPalatal movementXISpinal accessoryShoulder shrugXIIHypoglossalTongue protrusion

CN, cranial nerve.

TABLE 11-2 Innervation of the Eye by Its Six Nerves Number and Name of Nerve

InnervationClinical Effects of Interruption of NerveEfferent

CN III Striated muscle: superior, Diplopia, eye abducted and

Page 3: Neurology

(oculomotor nerve)

medial, and inferior recti; inferior obliqueLevator palpebraeSmooth muscle: pupilloconstrictorCiliary muscle

turned downPtosis (paralysis of volitional lid elevation)Pupil dilated and fixed to lightLoss of lens thickening

CN IV (trochlear nerve)

Striated muscle: superior oblique

Diplopia, most severe on looking down and in; eye extorted; head tilted to side opposite paralyzed eye

CN VI (abducens nerve)

Striated muscle: lateral rectuus

Diplopia, most severe on looking to side of paralysis; eye turned in (adducted)

Carotid sympathetic nerve

Smooth muscle; superior tarsal and pupillodilator

Horner's syndrome (ptosis, miosis, hemifacial anhidrosis, vasodilation)Afferent

CN II (optic nerve)

From retinaBlindness

CN V (trigeminal nerve)

Corneal/conjunctival afferentsAnesthesia of cornea with loss of corneal reflex

Adapted from NMS Neuroanatomy. Malvern, PA: Harwal Publishing, 1988:219.

3. Sensory system. Regions of abnormal touch, pain (estimated by pinprick), temperature, vibration, and proprioception should be defined.

o a. Are the findings confined to one side of the body, the distribution of one or more dermatomes, or the territory of one or more peripheral nerves?

o b. Are the sensory changes found in a “stocking–glove” distribution? 4. Motor system

o a. The patient's strength should be defined as it pertains to individual muscles or groups of muscles. One conventional method of grading muscle strength for purposes of comparison and description is shown in Table 11-3.

o b. A pronator drift can be assessed by having patients extend their arms (palm upward) with their eyes closed. Any depression or pronation is significant.

o c. The presence of atrophy, fasciculations, spasticity, and rigidity should be noted.

o d. The patient's ability to perform rapid alternating and other complex maneuvers should be determined.

o e. The patient's stance and gait should be evaluated. 5. Coordination. Finger-to-nose and heel-to-shin testing should be performed.

The physician should look for Romberg's sign (i.e., swaying or falling when standing with eyes closed and feet close together).

6. Muscle stretch reflexes. The activity and symmetry of the brachioradialis (C5, C6), biceps (C5, C6), triceps (C7, C8), knee (L3, L4), and ankle (S1, S2)

Page 4: Neurology

reflexes should be determined. The presence of the Babinski response should be assessed with plantar stimulation.

TABLE 11-3 Medical Research Council of Great Britain Muscle Strength Grading Scale

Grade

Equivalent Patient Ability

5/5Normal ability4/5Ability to overcome gravity and some resistance imposed by the

examiner3/5Ability to overcome gravity only2/5Ability to move with gravity eliminated1/5Only a flicker of movement0/5Complete inability to move

C. Neurodiagnostic studies

1. Cerebrospinal fluid (CSF) evaluation o a. Indications. Study of the CSF can provide information about

intracranial pressure (ICP) and infection, bleeding, malignancy, and sterile inflammation within the central nervous system (CNS).

o b. Specific measurements and assays (1) Pressure. The opening pressure should be determined.

Pressure exceeding 180–200 mm H2O is abnormal when a patient is relaxed and in a lateral decubitus position.

(2) Protein. An elevated CSF protein level is a nonspecific indicator of inflammation or breakdown of the blood–brain barrier.

(3) Glucose. Hypoglycorrhachia (i.e., CSF glucose <40 mg/dL or a simultaneous CSF–blood glucose ratio of <0.6) suggests infection or sterile inflammation.

(a) Relatively common causes of hypoglycorrhachia include bacterial, fungal, or tuberculous infection; carcinomatous meningitis; and hypoglycemia.

(b) Less common causes include mumps, herpes simplex virus (HSV) or zoster infection, subarachnoid hemorrhage (SAH), sarcoidosis, syphilitic meningitis, and systemic lupus erythematosus (SLE).

(4) White blood cell (WBC) count. A WBC count exceeding 5 cells/mm3 is considered abnormal.

(a) Excess neutrophils suggest acute infection or, on occasion, sterile inflammation.

(b) Excess mononuclear cells suggest a viral infection, an indolent nonviral infectious process, or sterile inflammation.

(5) Blood. Blood may appear in the CSF as a result of the local trauma of a lumbar puncture (LP) or by CNS hemorrhage from multiple causes.

Page 5: Neurology

(a) A traumatic LP is suspected if gross blood exudes from the needle and then clears quickly, or if a large discrepancy exists between the numbers of red blood cells (RBCs) in the first drops of CSF obtained as compared with a later aliquot.

(b) A traumatic LP should not reveal a xanthochromic (yellow-tinged) CSF, because sufficient time would not have elapsed to cause breakdown of RBCs.

(6) Culture and Gram staining. These studies are indicated to evaluate the possibility of infection. A CSF Venereal Disease Research Laboratory (VDRL) test is indicated if CNS syphilis is a diagnostic consideration.

(7) Cytology. Cytologic examination is useful if malignancy is suspected.

(8) Intrathecal immunoglobulin production. This can be determined using the immunoglobulin G (IgG) index:

or through CSF electrophoresis, which reveals the presence of oligoclonal bands (discrete immunoglobulin aggregates). An elevated IgG index or oligoclonal bands are found in CNS inflammatory disorders such as multiple sclerosis (MS) and infections.

o c. Contraindications to the performance of an LP (1) A mass effect sufficient to cause distortion of the lateral or

third ventricles or a midline shift (2) A posterior fossa mass (3) A coagulopathy [e.g., a prothrombin time (PT) >3 seconds

over control or a platelet count of < 50,000/mm3] 2. Electroencephalography (EEG) and evoked potentials (EPs)

o a. EEG. is indicated in the evaluation of seizure disorders, encephalopathies, sleep disorders, and brain death. Prolonged video EEG monitoring is the gold standard for evaluating seizure problems that are difficult to diagnose or treat.

o b. EPs are repetitive afferent stimuli presented to the eye, ear, peripheral sensory nerves, or cerebral cortex that cause stereotypic wave forms that can be analyzed by computerized signal averaging methodology. (1) Visual, brain stem–auditory, and somatosensory EPs can detect lesions, which are often clinically silent, in the anatomic pathways subserving these sensory systems. (2) Motor EPs, which can be elicited by transcranial magnetic stimulation of the motor pathways, provide information about the integrity of the motor system.

3. Imaging studies

Page 6: Neurology

o a. Computed tomography (CT) scanning. Provides an image of the brain that allows definition of hemorrhage, edema, atrophy, mass lesions, and ventricular size.

(1) Intravenous contrast can be administered to define regions where the blood–brain barrier is not intact.

(2) CT scanning can also visualize the spinal cord and surrounding bony structures; transverse images are especially well represented.

o b. Magnetic resonance imaging (MRI) provides excellent anatomic depiction of the brain, especially the posterior fossa, and the spinal cord. Disease of the cerebral white matter is particularly well defined. Intravenous contrast can detect sites of a disturbed blood-brain barrier. Diffusion weighted imaging can detect regions of cerebral ischemia, as reflected by an area of decreased free water diffusion.

o c. Single-photon emission computed tomography (SPECT) provides an image-based estimate of cerebral blood flow after intravenous injection of a radioactive tracer.

o d. Noninvasive vascular studies (1) Duplex scanning provides an ultrasound image of the

extracranial carotid and vertebral arteries together with a Doppler description of flow patterns.

(2) Transcranial Doppler (TCD) defines intracranial large artery flow patterns.

(3) Magnetic resonance angiography (MRA) uses magnetic resonance technology to image the vascular anatomy.

(4) Computed tomography angiography (CTA) uses computed tomography technology with intravenous contrast administration to image the vascular anatomy.

o e. Angiography. The vascular anatomy is best defined with cerebral angiography. This study is useful for identifying an aneurysmal source of SAH, evaluating occlusive cerebrovascular disease (especially if surgery is contemplated), defining vasculitis, and assessing arteriovenous malformations.

4. Nerve conduction velocity (NCV) studies and electromyography (EMG) o a. NCV studies. Can place peripheral nerve disease into sensory,

motor, or sensorimotor categories; define primarily demyelinating or axonal dysfunction; and identify sites of conduction block. These distinctions help in making a diagnosis.

o b. EMG can differentiate problems affecting muscle so that they can be divided into broad categories such as denervation and myopathy.

(1) The distribution of the observed changes helps determine whether the problem is myotomal (i.e., limited to a few nerve roots) or diffuse.

(2) The pattern of wave forms provides information pertaining to ongoing muscular denervation and reinnervation.

II. Loss of ConsciousnessA. Syncope

Page 7: Neurology

1. Definition. Syncope refers to a transient loss of consciousness that typically follows insufficient blood supply to the brain for more than a few seconds.

2. Clinical signs. Patients are transiently unresponsive, with diminished muscle tone. A few generalized tonic spasms may occur, especially if patients are prevented from lying down.

3. Etiology. Cardiac and circulatory causes of syncope are discussed in Chapter 1 IX. Most patients with syncope have a cardiac or circulatory basis for the event, although neurologic conditions need to be considered if the diagnosis remains elusive.

o a. Circulatory disturbances. are particularly common. (1) Vasovagal syncope, which is often seen in young people, is

commonly associated with emotional stress, fear, or pain. (2) Postprandial syncope, which frequently affects the elderly,

often occurs following meals in which alcohol has been consumed.

(3) Syncope can occur in diverse settings that have in common a preceding Valsalva or straining maneuver that decreases venous return and promotes parasympathetic tone.

o b. Cardiac output disturbances, other than arrhythmia or mechanical obstruction, should be considered.

(1) Vasodepressor (neurocardiogenic) syncope is caused by undue stimulation of afferent cardiac mechanoreceptors because of cardiac distention or strenuous contractions. This causes a decrease in sympathetic activity and an increase in parasympathetic activity that leads to vasodilatation, bradycardia, and subsequent hypotension.

(2) Carotid sinus hypersensitivity can lead to bradyarrhythmias and hypotension. Because carotid sinus hypersensitivity is present in many older men, it should be considered responsible for syncope only if other causes have been excluded.

o c. Hypoglycemia causes a lack of nutrient supply to the brain and can lead to syncope. Hypoglycemia as a cause of syncope is particularly likely in patients with type 1 (insulin-dependent) diabetes. Therapy involves the administration of glucose.

o d. Neurologic disorders are relatively uncommon causes of syncope. Several conditions are important because they can lead to unresponsiveness and are therefore often considered during the evaluation of a patient with transient unresponsiveness.

(1) Seizures (see also IX A–B) are a cause of unresponsiveness that must be differentiated from syncope.

(a) Patients are rarely limp during seizures. Many seizures cause intermittent, relatively rhythmic limb contractions (clonic activity) or sustained limb extension (tonic activity).

(b) Atonic seizures are rare in adults but do cause sudden collapse. Absence seizures, conversely, do not result in a fall.

(c) Because some patients with syncope can exhibit involuntary movements, the possibility of a primary seizure is a consideration. However, in most cases, the

Page 8: Neurology

tonic or myoclonic activity tends to occur several seconds after consciousness is lost and merely reflects cerebral hypoperfusion, not a primary seizure disorder.

(2) SAH can transiently increase ICP, compromising global cerebral perfusion. Clues to the diagnosis include persistent headache, meningismus, and papilledema.

(3) Basilar artery migraine (a unique type of migraine with aura) is a rare cause of unresponsiveness. A history of recurrent headache, recurrent episodes of unresponsiveness, and associated symptoms (e.g., visual distortion and dizziness) should lead to consideration of this condition. In most instances, other causes of vertebrobasilar ischemia should be sought before assigning a diagnosis of basilar artery migraine.

(4) Narcolepsy can cause episodes of sleep or cataplexy that can be mistaken for syncope.

o e. Psychogenic unresponsiveness can be associated with anxiety, panic attacks, or hyperventilation, as well as somatoform (conversion) disorder.

4. Diagnosis. A history and physical examination often can provide clues to the proper diagnosis. If no clues are evident, it is generally appropriate to proceed with a cardiovascular evaluation as outlined in Chapter 1 IX D.

o a. Neurologic testing frequently includes an EEG. Rarely is a CT scan diagnostic. In some patients, an MRI or an imaging study of the vascular system can be informative.

o b. Upright tilt testing, possibly with isoproterenol infusion, can provide evidence for a diagnosis of vasodepressor (neurocardiogenic) syncope, especially if the characteristic hemodynamic changes occur in less than 15 minutes without isoproterenol infusion.

5. Therapy. Treatment depends on the underlying diagnosis. First-line therapy of neurocardiogenic syncope is treatment with β-adrenergic blockers.

B. Coma

1. Definition. Coma is a state in which a patient is unresponsive to environmental stimuli and unable to communicate in any manner. Coma is associated with extensive structural or physiologic damage to both cerebral hemispheres or to the ascending RAS in the diencephalon, mesencephalon, or pons.

2. Etiology. The many causes of coma can be broadly grouped as shown in

Online Table 11-4.ONLINE TABLE 11-4 Causes of Coma

CategoryPossible Etiologic FactorsSupratentorial (hemispheric) lesions

Epidural or subdural hematomaIntraparenchymal hemorrhageLarge ischemic infarctionTumorAbscessTrauma

Page 9: Neurology

Infratentorial lesionsPontine or cerebellar hematomaBasilar artery thrombosisIschemic cerebellar infarctionTumorAbscess

Diffuse diseases, metabolic disorders, and toxins

Subarachnoid hemorrhageMeningitisEncephalitisHydrocephalusDrugs (e.g., narcotics, alcohol, barbiturates, and benzodiazepines)Hypo- or hyperglycemiaIschemic or hypoxic encephalopathyHypercarbiaMyxedemaHypothermiaHepatic or renal failureThiamine deficiencyPsychogenic

3. Approach to the patient. Complete and rapid assessment is critical for optimal care.

o a. Patient history. The physician should ascertain the following information from someone close to the patient:

(1) Past medical status, especially if there is a preexisting neurologic, cardiac, pulmonary, hepatic, or renal condition

(2) Prescription and over-the-counter drugs used by the patient (3) History of drug abuse, if applicable (4) Recent patient complaints (5) Details regarding the site where the patient was found (e.g.,

presence of empty drug vials, evidence of a fall)o b. Physical examination. The examination should be thorough.

Extremes of blood pressure, pulse, or temperature, abnormal breathing patterns, evidence of head or neck trauma, and the presence of meningismus should be noted carefully. The skin should be inspected for signs of trauma or needle tracks. Special attention should be directed to the patient's:

(1) Pupils. Pupillary size and reactivity are dependent on sympathetic and parasympathetic innervation. Brainstem reflexes such as the pupillary reaction to light offer clues to the location of the lesion responsible for the coma.

(a) Large, nonreactive pupils result from the disruption of the parasympathetic portion of the third cranial nerve, but may also be seen with barbiturate overdose.

(b) Small, reactive pupils result from the disruption of the sympathetic pupillodilatory impulses that arise in the hypothalamus and course caudally through the periaqueductal gray matter and cervical spinal cord

Page 10: Neurology

before traveling rostrally with the internal carotid artery toward the eyes.

(c) Pinpoint pupils that are nonreactive to light may be seen with narcotic overdose.

(2) Ocular motility. Analysis of ocular motility allows assessment of damage to the brainstem and the cranial nerves that control eye movement.

(a) The eyes should first be examined in the resting position for spontaneous motion of the eyeballs. Although the eyes of comatose patients may move spontaneously, they do not fixate or track in a purposeful manner.

(b) If the eyes are immobile, movement can be elicited through the vestibulo-ocular reflex by moving the patient's head side to side (the “doll's eyes” or oculocephalic maneuver) or by elevating the patient's head 30 degrees and irrigating the external auditory canal with ice water. The former should only be performed after a cervical injury is ruled out.

(i) Conjugate deviation of the eyes bilaterally implies intact brainstem circuitry.

(ii) Failure of an eye to abduct in response to these maneuvers implies dysfunction of pontine structures or sixth nerve compromise.

(iii) Failure of an eye to adduct implies dysfunction of the medial longitudinal fasciculus or oculomotor nucleus or nerve.

(iv) The presence of conjugate nystagmus away from the side of ice water irrigation suggests psychogenic coma.

(3) Motor functions. Quadriparesis, hemiparesis, or monoparesis may occur in comatose patients.

(a) Quadriparesis and flaccidity suggests pontine or medullary compromise or a high cervical spinal cord insult.

(b) Decorticate posturing (i.e., leg extension with flexion of the arm, wrist, and fingers) can be unilateral or bilateral and suggests a hemispheral or diencephalon lesion.

(c) Decerebrate posturing (i.e., leg and arm extension) also can be unilateral or bilateral and suggests midbrain or pontine compromise.

4. Clinical features. Once global brainstem dysfunction has developed differentiation between Supratentorial and Infratentorial causes of coma cannot be made without diagnostic testing unless a history and serial observations of the patient's clinical course can be documented.

o a. Supratentorial causes of coma. are often characterized by pathologic processes that result in swelling of a cerebral hemisphere. (1) This mass effect causes a midline shift of the affected hemisphere toward the contralateral side, compression of the ipsilateral third nerve

Page 11: Neurology

as it courses near the medial temporal lobe (uncus), herniation of the medial temporal lobe below the tentorial notch (uncal herniation), distortion of the mesencephalon, and herniation of the cingulate gyrus under the midline falx (subfalcial herniation).

(2) Typically, there is a progressive clinical deterioration characterized by increasing unresponsiveness, development of a third nerve palsy ipsilateral to the swollen hemisphere, and, ultimately, midbrain compromise (reflected by bilaterally nonreactive, dilated pupils).

o b. Infratentorial causes of coma can be suspected if ataxia, multiple asymmetric cranial nerve palsies, and unilateral or bilateral limb weakness or sensory loss develop before the development of more global, severe impairment of brainstem function (characterized by nonreactive pupils, absent ocular motility, and absent corneal and gag reflexes).

o c. Diffuse, toxic, or metabolic causes of coma can be suspected if pupillary responses are intact, ocular motility is preserved, corneal reflexes can be elicited, a gag reflex is present, and limb movement in response to noxious local stimuli is observed. If pupillary responsiveness remains even when other brainstem and limb function is lost, a metabolic cause of coma should be considered.

o d. Psychogenic coma should be suspected if the patient has a history of psychiatric disease or if the findings on physical examination are nonphysiologic. Examples of nonphysiologic responses in a “comatose” patient include:

(1) The presence of nystagmus when the patient's ears are irrigated with ice water

(2) Adversive head and eye movements (3) Failure of the patient's arm, when held by the examiner over

the patient's face, to fall on the face when released by the examiner

(4) Resistance to having the eyelids opened 5. Therapy. Ideally, care of the comatose patient is intertwined with the initial

assessment and the development of etiologic hypotheses.o a. Initial therapy. The “A, B, C's” Maintaining an adequate airway,

optimal ventilation, and appropriate blood pressure are priority concerns.

(1) If cervical fracture is a possibility, immobilization of the neck is of great importance.

(2) Endotracheal intubation may be indicated to protect the airway.

(3) Blood samples for a complete blood count (CBC), electrolytes, glucose, renal and liver function studies, coagulation profiles, blood gases, and toxicology should be obtained.

(4) Intravenous thiamine (100 mg), one ampule of D50W, and Naloxone (0.4 mg) are often administered. Flumazenil can be given if benzodiazepine or hepatic coma is suspected.

o b. Management

Page 12: Neurology

(1) Imaging. If the patient's general medical condition permits, and if the cause of coma is not clearly cerebral anoxia after cardiopulmonary arrest or a drug overdose, most patients should have a brain CT scan to define the presence of an intracranial mass, cerebral edema, or hydrocephalus. Further management depends on the etiology of the coma.

(2) ICP evaluation and management (a) ICP evaluation. Consideration of the patient's ICP is

intimately tied to the evaluation of coma. The intracranial cavity has a finite volume and compliance.

(i) Normally, modest volume additions to the intracranial contents (e.g., from a small intraparenchymal hematoma) cause only a small rise in ICP.

(ii) With progressive incremental increases to the intracranial volume (e.g., from massive cerebral edema, a hematoma, or a tumor), the intracranial compliance decreases and the ICP markedly increases.

(iii) Because cerebral perfusion pressure is the result of the mean arterial pressure minus the ICP, an excessive rise of the ICP is associated with impaired cerebral perfusion and progressive neurologic deterioration.

(b) ICP management. If a pathologic process associated with elevated ICP is suspected, emergency management should include steps to decrease the pressure, or at the very least, avoid increasing it. If possible, the cerebral perfusion pressure, or CPP, where CPP = MAP – ICP, should be kept at greater than 60 mm Hg and the ICP at less than

P.524

20 mm Hg. Optimal management of increased ICP often requires direct ICP monitoring as well as determination of hemodynamic parameters.

(i) Patients can be hyperventilated with an Ambu bag before intubation. Intubation and endotracheal suctioning should be performed carefully to minimize elevation of ICP.

(ii) Fever and agitation should be minimized. (iii) The patient's head should be elevated 30

degrees and kept in midposition to optimize venous drainage.

(iv) Osmotic therapy is used to dehydrate the brain and decrease the ICP. Patients are kept euvolemic, and intravenous mannitol or

Page 13: Neurology

hypertonic saline is administered to achieve a hyperosmotic state.

(v) Durotomy and hemicraniectomy can be used to decompress swollen brain.

(c) Prognosis. The prognosis of coma is generally related to the cause of the coma, the depth of the coma, and the duration. In one series, the probability of good or moderate recovery was only 2% once the patient had remained in coma for 14 days.

C. Vegetative state

1. Definitions o a. The vegetative state is characterized by the unawareness of self or

external stimuli. Patients cannot interact with others in a meaningful fashion. Autonomic functions are relatively well maintained, and a sleep–wake cycle exists. Patients can survive with medical and nursing support.

o b. A persistent vegetative state is defined as a vegetative state that persists for at least 1 month after the initial brain insult. If, with continued observation (usually 3 months for nontraumatic injury), there is no meaningful recovery, the likelihood of functional recovery can be judged to be nil; the patient can be said to be in a permanent vegetative state.

2. Therapy. The family and physician should determine the level of treatment appropriate for the patient in a persistent vegetative state.

D. Brain death

1. Definition. Death is recognized as occurring when there is irreversible cessation of all brain function. A brain insult sufficient to cause complete loss of cerebral function should be documented, if possible.

2. Approach to the patient o a. Physical examination. Patients are completely unresponsive to

external visual, auditory, and tactile stimuli and are incapable of communication in any manner.

(1) Pupillary responses are absent, and eye movements cannot be elicited by the vestibulo-ocular reflex or by irrigating the ears with cold water.

(2) The corneal and gag reflexes are absent, and there is no facial or tongue movement.

(3) The limbs are flaccid, and there is no movement, although primitive withdrawal movements in response to local painful stimuli, mediated at a spinal cord level, can occur.

o b. Apnea test. Patients have no respiratory function. An apnea test should be performed to ascertain that no respirations occur at a PaCO2 level of at least 60 mm Hg. The oxygenation should be maintained as the PaCO2 is allowed to rise. The inability to develop respiration is consistent with medullary failure.

Page 14: Neurology

o c. Exclusionary criteria. A diagnosis of brain death cannot be made in the setting of drug intoxication, hypothermia (defined as a core temperature of <32°C), or severe hypotension (i.e., shock).

o d. Confirmatory tests. These tests are usually not necessary to diagnose brain death but can be used if doubt exists or if local statutes require them.

(1) An EEG does not demonstrate any physiologic brain activity.

(2) Tests to assess cerebral blood flow fail to show cerebral perfusion.

o e. Period of observation. Periodic evaluation is necessary before a diagnosis of brain death can be made, unless there is gross evidence of a nonsurvivable insult to the brain.

(1) Two evaluations (6 to 12 hours apart) are usually sufficient to support a diagnosis of brain death.

(2) In the presence of anoxic brain damage, 24 hours of observation are appropriate before declaring brain death.

III. Alteration in BehaviorA. Delirium

1. Definition. Delirium is a disorder of brain function affecting behavior and causing impaired attention and cognition, motor hyper- or hypoactivity, altered sleep–wake cycles, and altered states of arousal. It is often acute, reversible, and secondary to a medical or neurologic disorder.

2. Etiology. Generalized or focal causes of cerebral dysfunction are potential causes of delirium.

o a. Generalized brain dysfunction. Causes include the following: (1) Drugs, including anticholinergics, antiparkinsonians,

analgesics, Cimetidine, Digoxin, benzodiazepines, antidepressants, and illicit substances, may produce delirium. Withdrawal from alcohol, barbiturates, and benzodiazepines is associated with delirium as well. The serotonin syndrome consists of delirium and disordered motor and autonomic function; it results from overstimulation of serotonin receptors. The neuroleptic malignant syndrome causes delirium in association with fever, rigidity, tremulousness, and occasionally myoglobinuria.

(2) Metabolic alterations, including hypoxia, hypercarbia, hyponatremia, uremia, hepatic failure, hyperglycemia, hypoglycemia, fever, dehydration, hypercalcemia, myxedema, hyperthyroidism, porphyria, anti-thyroid antibodies (Hashimoto's encephalopathy), and thiamine and niacin deficiencies, can cause delirium.

(3) Diffuse insults to the brain such as meningitis, encephalitis, fat emboli, and disseminated intravascular coagulation (DIC) are associated with cognitive impairment.

(4) Nonconvulsive status epilepticus, including absence or complex partial seizures, may cause delirium. Postictal patients may also be delirious.

Page 15: Neurology

(5) Systemic infections such as a urinary tract infection, pneumonia, or sepsis.

o b. Focal cerebral disease. Differentiating a global cerebral disorder from a focal brain disease that also may cause altered behavior presents a clinical challenge. For example, focal brain disease can cause a subtle aphasia, which may be misinterpreted as delirium. Appropriate laboratory and neurodiagnostic tests should be performed based on the clinical presentation.

(1) Focal cerebral disease, typically caused by stroke, involves the nondominant temporoparietal area, frontal lobes, head of the caudate nucleus, thalamus (the top-of-the-basilar syndrome), or occipital lobes, which may cause blindness. Patients who have a focal cerebral disease may be agitated and experience hallucinations.

(2) Mass lesions also can cause a confusional state, especially if they are located in the frontal lobes.

3. Therapy. The aim of therapy is identification and treatment, when possible, of the causes of delirium. An offending agent may have to be withdrawn. Adequate nutrition should be maintained and the safety of the patient ensured. If necessary, sedation with a low dose of haloperidol or a newer antipsychotic such as Risperidone, Quetiapine, or Olanzapine can be helpful.

B. Dementia

1. Definition. Dementia can be defined as a global decline in cognitive function in clear consciousness (an acquired, progressive loss of cognitive function associated with an abnormal brain condition. It is not a feature of normal ageing. In essence, one should be extremely cautious in making the diagnosis of dementia in a delirious patient.

2. Etiology. The causes of dementia are many. Identification of a treatable condition masquerading as a degenerative process is critical. Other causes that may masquerade as dementia, including depression, Postictal state, acute confusional state (including drug induced) and psychogenic illness of old age.

o a. Some causes of dementia include Alzheimer's disease (the commonest cause in adults), Parkinson's disease, multiple cerebral infarcts, Huntington's disease, frontotemporal degeneration including Pick's disease, dementia with Lewy bodies, human immunodeficiency virus (HIV) infection, and Creutzfeldt-Jakob disease.

o b. Potentially treatable conditions that can manifest as dementia include depression (pseudodementia), normal pressure hydrocephalus (NPH), subdural hematoma, intracranial tumor, adverse (chronic) drug effects, thyroid disease (hypothyroidism), vitamin B12 deficiency, thiamine deficiency, syphilis, heavy metal, acute confusional state. Intoxication, conditions causing hypersomnia (e.g., sleep apnea syndrome), chronic meningitis, and Wilson's disease.

3. Alzheimer's disease. This condition is the most common cause of chronic dementia.

o a. Definition. Alzheimer's disease is a clinicopathologic entity characterized by progressive memory loss (the earliest symptom is forgetfulness of newly acquired information) and other cognitive

Page 16: Neurology

deficits. Onset commonly is late in life, although patients may be affected in middle age.

(1) The disease usually arises spontaneously, but genetic factors have been identified. Familial cases (fewer than 5% & typically autosomal dominant) have been associated with mutations of the genes for amyloid precursor protein, presenilin 1, and presenilin 2.

(2) There is an association between the age of onset of Alzheimer's disease and the apolipoprotein E genotype. Patients with the APOE4/4 genotype have the greatest risk for Alzheimer's disease at a given age.

o b. Prevalence. Alzheimer's disease is a burgeoning public health problem. It is estimated that 60%–80% of demented patients have Alzheimer's disease. The prevalence increases sharply with age, affecting 5%–15% of people over age 65 and about three times as many people age 85 and older (the fastest-growing segment of the population).

o c. Pathology. Although the cause and pathogenesis are unknown, Alzheimer's disease has a characteristic pathology consisting of intracellular neurofibrillary tangles (which contain Hyperphosphorylated tau protein, the density of tangles can correlate with dementia severity) and extracellular neuritic plaques (which contain a core of β amyloid) the tangles and plaques occur throughout the cortex in overlapping but separate distributions.

(1) The tangles are composed primarily of abnormally phosphorylated, microtubule-associated tau proteins.

(2) The amyloid protein, Aβ, is derived from amyloid precursor protein and is deposited in senile plaques and blood vessels. The gene for amyloid precursor protein resides on chromosome 21 and may be involved in familial cases.

(3) Associated pathologic processes disturb many neurotransmitters, particularly the cholinergic system.(there is loss of cholinergic neurons and loss of choline acetyltransferase activity throughout the cortex. Other neurotransmitters are also affected.

(4) Macroscopic changes include brain atrophy, enlarged ventricles. Hippocampal atrophy can be one of the 1st signs.

o d. Diagnosis (1) The clinical diagnosis of senile dementia of the Alzheimer's

type (SDAT) can be made if an otherwise alert patient exhibits progressive memory loss and other cognitive deficits such as disorientation, language difficulties, inability to perform complex motor activities, inattention, visual misperception, poor problem-solving abilities, inappropriate social behavior, and, occasionally, hallucinations.

(a) The intellectual decline should be present in two or more domains of cognition and be documented by clinical examinations such as the mini-mental state examination. The original examination was published in “Mini-Mental State.” A Practical Method for Grading

Page 17: Neurology

the Cognitive State of Patients for the Clinician. J Psychiatr Res 1975; 12(3):189–198. Table 11-5 shows sample test items and where the test currently may be obtained.

TABLE 11-5 MMSE Sample Items Orientation to Time   “What is the date?”Registration   “Listen carefully; I am going to say three words. You say them back after I stop.   Ready? Here they are …APPLE (pause), PENNY (pause), and TABLE (pause). Now repeat those words back to me.” [Repeat up to 5 times, but score only the first trial.]Naming   “What is this?” [Point to a pencil or pen.]Reading   “Please read this and does what it says.” [Show examinee the words on the stimulus form.]CLOSE YOUR EYES“Reproduced by special permission of the Publisher, Psychological Assessment Resources, Inc., 16204 North Florida Avenue, Lutz, Florida 33549, from the Mini Mental State Examination, by Marshal Folstein and Susan Folstein, Copyright 1975, 1998, and 2001 by Mini Mental LLC, Inc. Published 2001 by Psychological Assessment Resources, Inc. Further reproduction is prohibited without permission of PAR, Inc. The MMSE can be purchased from PAR, Inc. by calling (813) 968-3003.”

(b) Formal neuropsychologic testing can confirm the clinical impression and document progression of the disease. Tests that address recall (with or without cues) and delayed recall are especially sensitive for documenting early memory impairment.

(c) Other systemic and neurologic diseases that could produce cognitive decline should be absent.

(2) Differential diagnosis (a) Patients with pseudodementia (depression) can

exhibit many of the features of Alzheimer's disease. To complicate matters further, patients with Alzheimer's disease may present with depression. Identification of patients with pseudodementia is important, because treatment of the depression can restore cognitive function.

Page 18: Neurology

(i) A careful history and neuropsychological evaluation often can determine the proper diagnosis.

(ii) If doubt remains as to the role of depression in the clinical presentation, appropriate treatment for depression is warranted.

(b) Mild cognitive impairment (i) Individuals with mild cognitive impairment

have a memory impairment beyond that expected for normal aging. However, these individuals do not yet meet criteria for Alzheimer's disease.

(ii) People with mild cognitive impairment evolve to Alzheimer's disease at a rate of 10%–15% annually, compared with 1%–2% annually for normal individuals.

(c) Other types of dementia (i) Pick's disease is a frontotemporal dementia

characterized by personality changes, disinhibition, hyperorality, and frontotemporal atrophy on imaging studies. Hyperphosphorylated tau protein that accumulates in the cerebral cortex is associated with the disease.

(ii) Dementia with Lewy bodies is characterized by cognitive impairment that can fluctuate, hallucinations, and early parkinsonian features.

(iii) Frontotemporal dementia with Parkinsonism linked to chromosome 17 (FTDP-17) is characterized by the same behavioral characteristics as the frontotemporal dementia described for Pick's disease. It is associated with abnormal tau protein.

o d. Therapy (1) Medical therapy is useful in treating insomnia, agitation,

and depression. (a) In general, drugs should initially be given at a low

dose; the dose can be adjusted upward slowly as clinically indicated.

(b) Medications with a short half-life and few anticholinergic side effects are best tolerated.

(c) Cholinesterase inhibitors such as Donepezil, Galantamine, and Rivastigmine may improve cognitive and behavioral function.

(2) Day care centers (including day hospitals) and respite care are useful adjuncts to family supervision of the patient with Alzheimer's disease and other dementing disorders.

4. Normal pressure hydrocephalus (NPH) o a. Definition. NPH is a condition characterized by a triad of cognitive

impairment, urinary incontinence, and gait apraxia (i.e., impaired

Page 19: Neurology

ambulation without evidence of primary motor, sensory, or cerebellar dysfunction).

o b. Etiology. In most patients, the cause of NPH is unknown. However, NPH can follow SAH or meningitis, sometimes even years later.

o c. Diagnosis. NPH should be suspected in patients who present with the clinical features noted in III B 4 a. The following tests may help confirm the diagnosis.

(1) Imaging studies (a) CT or MRI reveals ventricular enlargement with

relatively little cortical atrophy (Figure 11-2). (b) Cisternography involves injecting a radionuclide

into the lumbar thecal sac and then taking serial determinations of the flow pattern of the radioactive bolus. In the presence of NPH, cisternography demonstrates persistent activity of the radionuclide in the lateral ventricles after 48 hours

View Figure

FIGURE 11-2 A nonenhanced computed tomography (CT) scan showing hydrocephalus consistent with normal pressure hydrocephalus (NPH). Note the lack of cortical atrophy (sulcal effacement).

(2) ICP monitoring for 24–48 hours can reveal transient pressure increases, if the diagnosis is in doubt.

o d. Therapy. Insertion of a ventriculoperitoneal shunt can improve the patient's condition, especially if performed within 6 months of the onset of the problem.

5. Creutzfeldt-Jakob disease. This progressive, degenerative illness is caused by prions (i.e., infectious proteinaceous particles) and is associated with a spongiform encephalopathy.

o a. The gene for the prion protein is on chromosome 20; approximately 10% of cases are hereditary. Illness can develop because of infection or somatic and germ cell mutations. The CSF 14-3-3 protein is a marker

Page 20: Neurology

for Creutzfeldt-Jakob disease. “Mad cow disease” probably represents transmission of prion disease from infected cows to humans via ingestion of bovine food products.

o b. Patients may exhibit myoclonus. The EEG often demonstrates periodic discharges and an abnormal background rhythm.

o c. Death usually occurs within several months of the onset of the disease.

IV. HeadacheMany patients are concerned that their headaches are caused by a life-threatening condition such as a brain tumor. Fortunately, this is rarely the case, but complaints of headache, which are extremely common, always deserve further evaluation.A. Etiology

1. Non-neurologic causes. Before assuming that cephalic discomfort is caused by an intracranial disorder, the physician should consider the possibility of a non-neurologic cause. Disorders of the head and neck such as sinus disease, glaucoma, dental infections, temporomandibular joint (TMJ) disease, ear pathology, muscular injury, or cervical spine problems can cause headache.

2. Intracranial stimulation of pain-sensitive structures. Problems that affect the meninges or distort the larger blood vessels cause pain

3. Life-threatening causes

o a. An intracranial mass causes a headache that typically develops insidiously and progressively worsens.

(1) Clinical features. The pain is unlike any the patient has experienced and may awaken the person from sleep. Occasionally, the headache is worse early in the day. With time, associated symptoms (e.g., nausea, vomiting, exacerbation with lifting and straining) can develop. On examination, evidence of focal CNS disease is typically apparent.

(2) Therapy. Treatment is directed at the underlying lesion.o b. A “sentinel” SAH causes the apoplectic onset of headache in

previously healthy individuals or the sudden occurrence of headache that is of unique character in a chronic headache sufferer (see also VIII C 1).

(1) Clinical features. The possibility that a headache is a result of SAH is strengthened if the cephalic discomfort cannot be easily attributed to any of the usual causes of head pain. No neurologic findings may be present on examination, and meningismus may be absent.

(2) Diagnosis. Given the potential seriousness of the condition, patients should have a cranial CT scan. If this is unrevealing, an LP documents the presence of subarachnoid bleeding.

B. Headache syndromes

1. Migraine

Page 21: Neurology

o a. Etiology. The cause of migraine is unknown, but several common precipitants have been observed.

(1) A family history of migraine often exists. (2) Headaches can be related to stress, altered sleep patterns,

menses, oral contraceptives, alcohol use, caffeine withdrawal, monosodium glutamate (MSG) intake, and various foodstuffs (e.g., chocolate, nuts, aged cheeses, and meats containing nitrates).

(3) Migraine can develop after seemingly minor head trauma; recognition and treatment may prevent prolonged disability.

o b. Pathophysiology. Hypotheses center on the idea that a migraine attack is brought on by neurovascular disturbances.

(1) The classic vasospasm–vasodilation theory arose from clinical observations. Recent data suggest that oligemia, secondary to a slowly spreading area of neuronal depolarization (the cortical depression of Leão), occurs during a headache prodrome and persists into the headache phase. Hyperemia occurs subsequently and can persist after the headache subsides.

(2) Current theory maintains that dysfunction of the trigeminovascular system, resulting in the perivascular release of substance P and other neurotransmitters and inflammatory markers, leads to migraine.

o c. Migraine syndromes (1) Migraine without aura (common migraine) is an intermittent

syndrome characterized by generalized or hemicranial pulsatile cephalic discomfort. Nausea, vomiting, photophobia, phonophobia, and anorexia may accompany the headache.

(2) Migraine with aura (classic migraine) presents with an aura, often a vivid visual array of colors in a geometric pattern involving one visual hemifield.

(a) The throbbing headache is often contralateral to the visual display, and nausea, vomiting, photophobia, phonophobia, and anorexia may be present.

(b) Migraine with aura also can be associated with transient neurologic deficits such as visual field deficits and hemisensory loss.

(c) On very rare occasions, stroke is a complication of migraine.

o d. Therapy. Treatment should first involve removal of inciting agents when possible.

(1) Abortive therapy for migraine (a) Ergotamine, available in oral, sublingual, nasal, and

suppository forms, is a serotonin (5-HT1)–receptor agonist that decreases substance P release at the trigeminovascular junction. Intravenous ergotamine [dihydroergotamine (DHE 45)] also has proved to be efficacious; pretreatment with metoclopramide or prochlorperazine prevents nausea

Page 22: Neurology

(b) Aspirin, nonsteroidal anti-inflammatory drugs (NSAIDs), and isometheptene can abort a migraine. Analgesics may be administered for symptomatic relief as well.

(c) Triptans, a family of serotonin 5-HT1–receptor agonists, are effective.

(2) Prophylactic measures include drug regimens and changes in patient behavior referable to headache precipitants.

(a) Medications such as β-blockers, tricyclic antidepressants, calcium channel blockers, NSAIDs, gabapentin, topiramate, or Valproic acid may be used to prevent migraines. Prophylactic medications, although of seemingly diverse types, all have a tendency to alter CNS serotonin activity.

(i) The choice of medication is guided, in part, by the need to avoid or exploit a particular drug action (aside from the antiheadache effect).

(ii) Initially, a low dose should be administered, and the therapeutic benefits and undesirable side effects should be monitored as the dose is increased. The dose can be increased until either a beneficial response is achieved or adverse side effects develop. A maximal dose is best maintained for several weeks before concluding that an agent is not effective.

(b) Biofeedback therapy may enable patients to lessen migraine events by helping them deal more effectively with stress.

2. A muscle contraction, or tension, headache is characterized by a band-like discomfort about the head.

o a. Clinical features. This type of headache often develops during the course of the day and may be associated with emotional stress. Posterior cervical and occipital muscles are often tender and may be in spasm. The distinction between this type of headache and migraine without aura can be difficult.

o b. Therapy. Treatment involves reassurance, NSAIDs, muscle relaxants, moist heat, and, on occasion, antidepressant drugs and psychotherapy.

3. Chronic daily headache o a. Etiology. Patients with migraine or tension headache can develop

chronic daily headaches spontaneously or as a result of excessive use of analgesics or ergotamines.

o b. Therapy. Treatment consists of withdrawal from excessive medications. Intravenous DHE 45 given for 2–3 days can help break the headache cycle. Prophylactic migraine agents can help prevent a headache recurrence.

4. Cluster headache o a. Clinical features. Cluster headaches are severe periorbital headaches,

30–90 minutes in duration, that occur once or several times daily over a period of several weeks or months. The unilateral pain may be

Page 23: Neurology

accompanied by ipsilateral lacrimation, conjunctival injection, nasal congestion, and Horner's syndrome. The typical patient is a middle-aged man. Patients with cluster headaches often pace, as opposed to migraineurs, who seek quiet, dark places.

o b. Therapy (1) Abortive and symptomatic treatment includes the

administration of 100% oxygen, ergotamines, analgesics, or sumatriptan.

(2) Prophylactic therapy incorporates lithium, calcium channel blockers, or corticosteroids.

5. Temporal (giant cell) arteritis (see also Chapter 10 XII D 1)o a. Clinical features. Patients over the age of 50 years who complain of

a headache centered about one temple or located in the occipital area should be evaluated for giant cell arteritis. Associated symptoms include visual disturbances, jaw claudication, fever, arthralgias and myalgias, and weight loss. Polymyalgia rheumatica (PMR) is also present in approximately 50% of patients with giant cell arteritis.

o b. Diagnosis. The erythrocyte sedimentation rate is typically greater than 40 mm/hour, and the C-reactive protein level is elevated. Biopsy of a temporal artery confirms the diagnosis.

o c. Therapy. Corticosteroid treatment can bring rapid relief. 6. Benign (idiopathic) intracranial hypertension (pseudotumor cerebri) has no

known cause but is associated with obesity, pregnancy, oral contraceptives, SLE, cranial venous sinus thrombosis, and a host of other conditions.

Clinical features. The development of a relatively constant, generalized headache in patients with a clear sensorium, papilledema, and an otherwise normal neurologic examination is suggestive of benign intracranial hypertension. Visual obscurations can occur, and visual loss is the most serious complication.

o b. Diagnosis. The diagnosis is suggested by a CT or MRI scan, which is normal. The diagnosis can be confirmed by finding an elevated CSF opening pressure and an otherwise normal CSF analysis.

o c. Therapy (1) Visual acuity and fields should be monitored. (2) Serial LPs can relieve the syndrome. (3) Corticosteroids, acetazolamide, or furosemide may be

administered. (4) Refractory disease has been managed with lumboperitoneal

shunting of CSF or optic nerve sheath fenestration. 7. Trigeminal neuralgia (tic douloureux) is a syndrome that most often is

idiopathic but has been associated with MS, neoplasia, and vascular “loops” that impinge on the trigeminal nerve.

o a. Clinical features. Lightning-quick, severe facial pain, often associated with a trigger point, is suggestive of trigeminal neuralgia. The painful jabs are usually restricted to one or two divisions of the trigeminal nerve. There is no loss of facial sensation.

o b. Therapy. Therapeutic modalities include carbamazepine, Baclofen, gabapentin, surgical intervention, and stereotactic radiation therapy.

Page 24: Neurology

8. Indomethacin-responsive headaches are characterized by severe, unilateral pain that may be relieved by treatment with indomethacin.

o a. Chronic paroxysmal hemicrania. is characterized by painful, multiple attacks (up to 40 daily) that last from 2 minutes to 2 hours with nocturnal awakenings and associated autonomic features. Alcohol can precipitate the attacks.

o b. Episodic paroxysmal hemicrania is characterized by painful, multiple attacks (6–30 daily) that last up to 30 minutes and are associated with nocturnal awakenings and other autonomic features. Remissions lasting months to years can occur.

9. Low pressure–volume headache is characterized by a headache that substantially worsens when in the upright position and conversely improves with a supine posture.

o a. Etiology. Postural headaches typically occur after LP but can develop spontaneously or in association with a defect in the integrity of the dura.

o b. Diagnosis. MRI shows generalized dural enhancement. The cerebellar tonsils may have descended below the level of the foramen magnum, and the brain stem may be “kinked.”

o c. Therapy. Treatment consists of a blood patch (injection of autologous blood into the lumbar epidural space).

V. WeaknessMany disorders can cause weakness. To pinpoint the causative disorder, the physician must first determine which part of the nervous system is diseased.A. Anatomic and functional approachWeakness can result from dysfunction at various points in the central (CNS) or peripheral nervous system (PNS). Specific localization depends upon recognition of the pattern of weakness and associated findings.

1. CNS disorders. Dysfunction of the pyramidal tracks or upper motor neurons is classically associated with “upper motor neuron signs” and “pyramidal weakness.”

o a. Upper motor neuron signs consist of increased reflexes, up-going toes, and increased tone (spasticity).

o b. Pyramidal weakness refers to a pattern whereby the extensors are weaker than the flexors in the upper extremity, whereas the flexors are weaker than the extensors in the lower extremity.

2. PNS disorders. Dysfunction of the motor unit (lower motor neurons and the muscle fibers they innervate) is associated with “lower motor neuron signs” and “site-specific weakness.”

o a. Lower motor neuron signs refer to reduced or absent reflexes, down-going or mute toes, and decreased tone, as well as atrophy and fasciculations (involuntary muscle twitches).

o b. Site-specific weakness refers to the specific patterns associated with pathology along the various sites of the motor unit.

(i) Lower motor neuron disease such as polio may produce weakness at various sites.

(ii) Nerve root disease will produce weakness in a “myotomal” pattern akin to the dermatomal sensory loss.

Page 25: Neurology

(iii) Disorders of the neuromuscular junction generally produce proximal weakness.

(iv) Disorders of muscle (myopathies) also produce proximal weakness.

B. Specific patterns of CNS pathology by anatomic location(Each pattern should be associated with upper motor neuron signs and pyramidal weakness.)

1. Cortical (precentral gyrus) lesions may be distinguished by associated changes on mental status examination.

o a. Small lesions may be associated with monoplegia and possible sensory loss in same area.

o b. Large hemispheric lesions (e.g., MCA stoke) should produce a hemiparesis, hemisensory loss, and a homonymous hemianopia. Aphasia is commonly seen with left-sided lesions while neglect may be seen with right-sided lesions.

2. Subcortical lesions (in the deep white matter or posterior limb of the internal capsule) classically produce a hemiparesis with the pattern of face=arm=leg, with possible hemisensory loss, but no visual changes and only rare mental status changes

3. Midbrain lesions (in the cerebral peduncles) are rarely seen in isolation, but may cause a subcortical pattern of weakness (face=arm=leg) with associated third nerve dysfunction or marked tremor.

4. Pontine lesions produce a subcortical pattern of weakness (face=arm=leg), possible hemisensory loss, and possible cranial nerve defects.

5. Medullary lesions are rarely seen in isolation; vascular lesions usually cause a contralateral hemiparesis sparing the face, ipsilateral tongue deviation and contralateral loss of vibration and proprioception (medial medullary syndrome).

6. Spinal cord lesions (involving the corticospinal tracts) cause ipsilateral weakness below the level of the lesion.

C. Specific patterns of PNS pathologySpecific patterns of PNS pathology by anatomic location. (Each pattern may have some lower motor neuron signs, but “site-specific” weakness.)

1. Motor neuron diseases (diseases of the anterior horn cell such as amyotrophic lateral sclerosis [ALS]) are pure motor disturbances with severe generalized weakness, wasting, hyporeflexia, and fasciculations.

2. Radiculopathy (nerve root disorders) produce weakness and sensory loss in the distribution of a nerve root (“myotomal” weakness and “dermatomal” sensory loss respectively) as well as radicular or shooting pain.

3. Plexopathies (e.g., brachial, lumbar, or sacral) produce weakness, numbness, hyporeflexia and pain in an entire limb.

4. Neuropathies generally produce distal numbness, weakness and reflex loss. 5. Neuromuscular junction disorders such as myasthenia gravis are pure motor

disorders characterized by proximal weakness.

Page 26: Neurology

6. Myopathies (disorders of muscle) are pure motor syndromes usually characterized by proximal weakness, with hyporeflexia only in very weak muscles.

VI. Disequilibrium and DizzinessA. AtaxiaAtaxia (gait instability) can be caused by cerebellar dysfunction or disorders of the motor or sensory system.

1. Cerebellar dysfunction. In addition to clumsiness and limb or truncal instability, as manifested by incoordination and impaired stance and gait, patients may exhibit hypotonia and ocular dysmetria.

a. Acute cerebellar dysfunction. (e.g., stroke, MS, neoplasia) typically manifests with unilateral findings.

o b. Causes of relatively symmetric subacute or chronic cerebellar dysfunction include alcoholic cerebellar degeneration; drug intoxication (e.g., phenytoin); cerebellar or spinocerebellar degenerations; and paraneoplastic, immune-mediated degeneration.

2. Motor or sensory disorders. Patients with strength in the 2/5–4/5 grade range (see Table 11-3) may appear clumsy, as may patients with a sensory neuropathy, dorsal root ganglion disease, posterior column dysfunction, or parietal lobe disease (parietal ataxia).

B. Dizziness

1. Approach to the patient o a. History. Obtaining a thorough history aids in patient evaluation. By

paying special attention to associated symptoms, the physician may be able to identify pathology of the inner ear, eighth cranial nerve, or CNS as the cause of the dizziness.

o b. Physical examination. The physician uses several screening techniques to search for evidence of structural disease.

(1) Nystagmus is present in many patients who have vertigo. The nystagmus can be observed spontaneously (when the patient gazes straight ahead, laterally, or vertically), or it may be elicited by using Frenzel lenses to block visual fixation, by having the patient shake his or her head during ophthalmoscopy, or by covering the contralateral eye during ophthalmoscopy (to decrease visual fixation).

(a) Horizontal or vertical nystagmus is characterized by a slow eye drift and a rapid shift in the opposite direction.

(b) Torsional nystagmus is characterized by a slow clockwise or counterclockwise rotation and a rapid movement in the opposite direction.

(c) The direction of nystagmus refers to the direction of the fast component of eye movement. Nystagmus is away from the affected ear (i.e., slow phase of eye

Page 27: Neurology

movement is toward the side of the lesion). It is often of a mixed horizontal and torsional type.

(2) Gaze mechanisms should be tested, including saccadic speed and accuracy, and the integrity of smooth pursuits. The presence of ocular dysmetria and visual acuity while shaking the head should be noted. Suppression of the vestibulo-ocular reflex (which may be tested by asking the patient to fixate on a target moving synchronously with the head) should be noted as well. The head thrust maneuver may demonstrate abnormal corrective saccades.

(3) The physician should check for the presence of benign paroxysmal positional vertigo (BPPV) by performing the Nylen-Bárány maneuver (i.e., by hyperextending the patient's neck and rotating the patient's head laterally as the patient is rapidly moved from a seated to a supine position). Rotary nystagmus with a linear component that appears after a several-second latency period suggests BPPV.

(4) Vertigo that worsens when pressure is applied to the tragus may suggest a perilymphatic fistula.

2. Selected causes of dizziness o a. Labyrinthitis (vestibular neuronitis). is an inflammatory condition of

the inner ear characterized by the acute onset of a spinning sensation (vertigo), exacerbated by movement, and associated with nausea and vomiting. The condition is usually felt to be viral or bacterial in origin.

o b. Ménière's disease is characterized by episodic vertigo accompanied by nystagmus, tinnitus, fluctuating hearing loss, and aural discomfort.

o c. BPPV occurs when the patient moves his or her head (e.g., by rolling over in bed, looking up, or standing up). This condition is usually idiopathic in nature but can be caused by trauma, viral or ischemic injury, or drug toxicity. A unique physical therapy technique that can be used to decrease symptoms of BPPV has been developed.

o d. Drug-induced vestibulopathy can be caused by Gentamicin, streptomycin, furosemide, and cisplatin.

o e. Eighth cranial nerve disease can be associated with hearing loss. Causes include acoustic neuroma, metastatic disease, vasculitis, and basilar meningitis associated with infectious and inflammatory processes.

o f. Lateral medullary syndrome (Wallenberg's syndrome) is due to occlusion of the vertebral or posterior inferior cerebellar artery.

(1) Patients often feel that the external world is “tilting” and complain of feeling propelled toward one side (usually the side of the lesion).

(2) Physical examination can indicate nystagmus (often horizontal–torsional and away from the lesion), ipsilateral ataxia, ipsilateral Horner's syndrome (characterized by ptosis, miosis, and anhidrosis), and ipsilateral facial sensory impairment with contralateral body sensory loss.

o g. Cerebellar disease can be caused by stroke, tumor, infection, or degenerative and inflammatory disease (see XVIII H). Paraneoplastic cerebellar degeneration is associated with anti-Yo antibodies (leading

Page 28: Neurology

to Purkinje cell loss) and anti-Ri (antineuronal) antibodies. Physical examination indicates ataxia and postural instability and many types of nystagmus. Opsoclonus is associated with anti-Ri antibodies.

o h. Brainstem lesions are typically associated with cranial nerve, motor, sensory, and cerebellar dysfunction.

o i. Other causes of dizziness include postural hypotension, hyperventilation, hypoglycemia, hypothyroidism, epilepsy, migraine, psychiatric disorders (e.g., depression, anxiety, and somatization disorders), and multisensory impairment (i.e., poor vision and positional sense).

3. Therapy o a. Dizziness may be relieved by treating the underlying disease. If the

underlying disease is unknown or otherwise untreatable, acute symptomatic therapy includes the use of meclizine, prochlorperazine, promethazine, topical scopolamine, and diazepam.

o b. Vestibular and gait exercises may help habituate the vestibular pathways to the abnormal influences of the disease.

o c. Chronic, poorly defined causes of dizziness can be managed by withdrawing the patient from drugs that affect the vestibular system. This action may allow a better reevaluation of the patient. Treatment with a low dose of a benzodiazepine and vestibular and gait exercises may be effective.

VII. Pain SyndromesWhen patients complain of pain, it is important to explore whether the pain is new-onset, long-standing, or intermittent, and whether it is static or progressive. Patients should be asked to identify exacerbating and alleviating factors as well as associated symptoms such as weakness or numbness.A. Pain originating from the lower back

1. Etiology o a. Diagnostic considerations include local muscular strain, traumatic

conditions of the spine, degenerative and inflammatory arthritides, neoplastic and infectious processes, and nerve root irritation.

o b. Neurologic dysfunction commonly develops from impingement on nerve roots by disk material, bony overgrowth, or thickened ligaments.

2. Disorders associated with low back pain o a. Sciatica. Denotes a syndrome of sharp pain radiating from the low

back to the buttock, down the back of the thigh to the calf, and, at times, over the bottom or top of the foot. Sciatica is commonly associated with disk herniation but can result from other conditions that irritate nerve roots L4, L5, or S1.

(1) Clinical features. Myotomal weakness and dermatomal numbness, along with appropriate muscle stretch reflex changes and pain that increases when the examiner raises the patient's extended leg, can occur.

(2) Diagnosis. Persistent symptoms and neurologic deficits warrant in-depth investigation, which commonly includes the use of lumbosacral radiographs, lumbar MRI, and EMG. CT

Page 29: Neurology

myelography may be indicated for better definition of the bony anatomy.

P.535

(3) Therapy. Acute low back pain should be evaluated carefully. Treatment should be directed at the underlying disorder if possible.

(a) Unless tumor or infection is the likely cause for sciatica, initial therapy should include avoidance of strenuous activity and treatment with NSAIDs. If patients show signs of a cauda equina syndrome (i.e., a flaccid bladder, rectal dysfunction, and bilateral lower motor neuron leg weakness), emergency surgery is usually required.

(b) Long-term therapy may be appropriate. As the acute syndrome remits, a regimen of back-strengthening exercises and weight loss (if applicable) can be helpful. Surgery (e.g., laminectomy) may be necessary to relieve persistent symptoms.

o b. Lumbar stenosis is another common condition resulting in low back pain. Patients often have a congenitally small lumbar canal. Over time, bony and ligamentous overgrowth and disk protrusions may further encroach on the neural and vascular contents of the canal and foramina.

(1) Clinical features. Patients develop pain and, occasionally, sensory loss and weakness with ambulation and prolonged standing, but they are relatively comfortable at rest. The neurologic examination of resting patients is often quite normal.

(2) Therapy. It is necessary to differentiate pain caused by lumbar stenosis from that caused by vascular insufficiency. If symptoms are severe, decompression surgery is often pursued for treatment of lumbar stenosis.

B. Pain originating from the neck

1. Etiology. Neck pain is often due to trauma (e.g., “whiplash”). Degenerative and inflammatory bone and disk disease can lead to nerve root irritation and is also a common source of neck pain. Local infection should be considered for acute symptoms; a neoplasm may be the culprit when subacute symptoms exist.

2. Disorders associated with neck pain o a. Cervical radiculopathy. Pain can radiate in a dermatomal pattern

with a parallel loss of sensation. Weakness may be in a myotomal distribution with a corresponding loss of muscle stretch reflexes.

Page 30: Neurology

o b. Arthritis. The cervical radiculopathy associated with cervical arthritis (spondylosis) can often be treated with a soft cervical collar, NSAIDs, muscle relaxants, and, occasionally, surgery.

C. Selected pain syndromes

1. Herpes zoster o a. Acute syndrome. “Shingles”. is a painful condition caused by the

activation of a latent herpes zoster infection, which affects the nerves that supply the skin. Although most common in the elderly, shingles can occur in immunosuppressed patients as well.

(1) Clinical features. The rash, characterized by painful vesicles on an erythematous base, may be preceded by discomfort. The pain often abates when the skin lesions heal but may persist and lead to postherpetic neuralgia.

(2) Therapy (a) Corticosteroids may lessen the risk of postherpetic

neuralgia in nonimmunosuppressed patients older than 60 years.

(b) Famciclovir decreases the duration of postherpetic neuralgia in immunocompetent patients.

(c) Acyclovir should be given to immunosuppressed patients to hasten recovery.

(d) Analgesics, tricyclic antidepressants, gabapentin, and topiramate treat pain during the acute phase of the illness.

o b. Chronic syndrome (1) Once the vesicles have healed, residual discomfort can be

managed with Gabapentin, topical Lidocaine, Pregabalin, and capsaicin ointment.

(2) Tricyclic antidepressants, carbamazepine and other anticonvulsant medications, vapo-coolant spray, transcutaneous electrical nerve stimulation, and, occasionally, surgical intervention may be effective.

P.536

2. Complex regional pain syndrome (reflex sympathetic dystrophy; sympathetic maintained pain). This chronic painful condition can occur idiopathically or develop after trauma to a limb. At times the severity of the initial injury can be quite trivial.

o a. Clinical features. The affected limb is painful, with altered sensory perceptions, vasomotor tone, and temperature. The skin is often discolored.

o b. Diagnosis. To make a diagnosis, other local conditions that can cause pain must be eliminated. A differential nerve block via infiltration of paravertebral nerves with varying doses of local anesthesia helps differentiate between local somatic nerve

Page 31: Neurology

hyperexcitability and the autonomic sympathetic dysfunction attributable to sympathetic mediated pain. Intravenous phentolamine blockade can also provide evidence for sympathetic mediated pain.

o c. Therapy. Treatment includes paravertebral sympathetic ganglion blocks and aggressive physical therapy. Administrations of tricyclic antidepressants, anticonvulsants, or topical capsaicin or Lidocaine are useful adjuvant therapies.

D. Chronic pain syndromesNonmalignant pain syndromes, when chronic, can be especially difficult to evaluate and manage.

1. Care should be taken to assess patients fully for remedial problems (e.g., lumbar stenosis). An interdisciplinary team approach is best.

2. Use of antidepressants and supportive psychotherapy, coupled with attempts to improve the patient's functional status, can be beneficial. In selected instances, chronic opiate therapy can be beneficial.

VIII. StrokeThis neurovascular disorder is the most common neurologic disease causing serious morbidity and mortality.A. Introduction

1. Disease of the vascular system can disrupt blood supply to the CNS, leading to neuronal dysfunction. Stroke syndromes can be broadly classified into predominantly ischemic or hemorrhagic processes. Asymptomatic lesions in either category of stroke syndrome can predispose patients to future disease.

2. Optimal management requires precise diagnosis for effective therapy.o a. General principles of management. Of symptomatic patients include

monitoring for signs of clinical deterioration, ensuring adequate oxygenation, maintaining euvolemia, avoiding extremes of blood pressure without excessively lowering high blood pressure, treating infection and fever, and using normal saline (rather than glucose solutions) for intravenous hydration. Efforts should be made to prevent deep venous thrombosis and decubiti, avoid aspiration, maintain nutrition, and attend to bowel and bladder function.

o b. Rehabilitation of patients with persistent neurologic defects is important.

(1) Multidisciplinary therapy involving physical, occupational, and speech therapy can be pursued on an inpatient or outpatient basis.

(2) Depression occurs frequently and may be treated medically.

B. Ischemic stroke

1. Introduction o a. Etiology. Causes include cardiogenic emboli, extracranial and

intracranial large artery disease, small artery disease, and various systemic and hematologic disorders.

o b. Clinical signs

Page 32: Neurology

(1) Impaired carotid territory circulation produces symptoms of contralateral weakness and sensory loss, aphasia or neglect syndromes and ipsilateral transient monocular blindness (amaurosis fugax). Typically, the weakness or numbness is greatest in the facial area, less pronounced in the arm, and still less pronounced in the leg.

(2) A diagnosis of vertebrobasilar disease is fairly certain if there is transient binocular blindness or other bilateral visual disturbances, diplopia, ataxia, quadriparesis, or vertigo associated with other neurologic symptoms.

o c. Therapy. New treatments for acute ischemic stroke are under investigation, driven by the evidence that there is at least a 3- to 6-hour “therapeutic window” during which intervention may lessen brain damage.

(1) Thrombolytic agents may be administered with an acceptable incidence of hemorrhagic infarction and intraparenchymal hematoma. A randomized, double-blind trial of intravenous recombinant tissue plasminogen activator (t-PA) administered no more than 3 hours after the onset of an acute ischemic stroke at a dose of 0.9 mg/kg (with 10% of the dose given as a bolus and the remainder infused over 1 hour) resulted in an improved clinical outcome at 3 months in patients treated with the active drug. Patients treated with t-PA were more likely to sustain a symptomatic intracerebral hemorrhage during the

P.537

first 36 hours; however, there was no significant difference in mortality between treated and untreated patients.

(a) Treatment with t-PA appears to be beneficial for patients with small or large artery occlusive disease or cardioembolic stroke. Because there is such a narrow window of opportunity for the administration of t-PA, rare patients destined to have a transient ischemic attack (TIA; see VIII B 2) will be inadvertently treated.

(b) Patients to be treated with t-PA must meet strict criteria to minimize the risk of hemorrhagic complications. In addition, no anticoagulants or antiplatelet agents should be administered for 24 hours after t-PA treatment, and blood pressure elevations should be treated. Contraindications include:

(i) A rapidly improving neurologic deficit or minor symptoms

(ii) A baseline CT scan showing evidence of intracranial hemorrhage

(iii) A systolic blood pressure >185 mm Hg or a diastolic blood pressure >110 mm Hg

Page 33: Neurology

(iv) A medication-induced or disease-related coagulopathy

(v) A history of hemorrhagic stroke, recent surgery, or another invasive procedure

(2) Although not yet “standard of care,” there is increasing experience with intra-arterial thrombolysis and several mechanical approaches to intra-arterial clot lysis for patients acutely presenting with large artery thromboembolic occlusions. A variety of imaging techniques, including CT and MR perfusion scans, MR diffusion, and CT or MR angiography may assist in patient selection.

2. Transient ischemic attacks (TIAs) are short-lived neurologic deficits, typically lasting minutes. A recent recommendation is that the maximal duration of a TIA be revised downward to 1 hour, in contradistinction to the classic definition of up to 24 hours. Symptoms are attributable to ischemia in the carotid or vertebrobasilar arterial distributions. The distinction between a TIA and a stroke is arbitrary, and even a brief symptomatic episode of cerebral ischemia, in conjunction with an abnormal diffusion weighted imaging (DWI) scan, might be construed as representing a stroke. Both warrant complete evaluation to determine the underlying pathophysiology and decrease the risk of subsequent ischemic events.

o a. Etiology. Although TIAs often result from atherosclerotic large vessel disease, other diagnostic possibilities deserve consideration, including cardiogenic emboli, aortic arch atherothrombotic emboli, and other large artery disorders such as dissection and fibromuscular dysplasia, small artery disease, hematologic disorders, and migraine. Other disease entities such as seizures, tumors, subdural hematomas, and MS sometimes masquerade as a TIA.

o b. Diagnosis. Diagnostic studies should include a CBC, syphilis serology, a coagulation profile, and a CT scan or MRI. A transthoracic or transesophageal echocardiogram and 24-hour ambulatory electrocardiography (ECG) monitoring may be indicated.

(1) A duplex examination of the extracranial carotid artery territory is often informative.

(2) TCD studies, MRA, and CTA can provide insight into the extracranial and intracranial arterial circulation.

(3) Conventional interventional angiography is the definitive test to outline the vascular anatomy.

(4) Patients with an unrevealing cardiac evaluation who are thought to have an embolus may benefit from a transesophageal echocardiogram.

(5) If clinical suspicion exists, an LP can be used to evaluate the possibility of CNS inflammation.

o c. Therapy. If atherosclerosis is suspected as the cause of transient cerebral ischemia, control of risk factors for atherosclerosis is essential (see Chapter 1 III A 2). An evaluation for coronary artery disease is also appropriate.

(1) Extracranial carotid artery disease. Carotid endarterectomy is superior to medical therapy in the prevention of ischemic stroke in patients experiencing a TIA ipsilateral to an

Page 34: Neurology

angiographically demonstrated 50%–99% stenosis at the internal carotid artery origin.

(a) Patients with less severe stenosis should be treated with aspirin. Clopidogrel or aspirin combined with slow-release dipyridamole are also effective in preventing stroke.

(b) Risk factor management pertinent to atherosclerosis is appropriate.

(2) Intracranial large artery disease. A TIA can result from large artery stenosis or occlusion. Recent evidence suggests that aspirin therapy is safer than, and as effective as, anticoagulation therapy for symptomatic intracranial large artery stenosis.

P.538

(3) Other causes of TIA. Therapy should be directed at the appropriate pathophysiologic process. For instance, anticoagulants are frequently used in patients with a source of cardiogenic emboli.

3. Cardiogenic embolic stroke o a. Etiology. The most common cause of embolic stroke is nonvalvular

atrial fibrillation. (1) Other conditions associated with cardiogenic emboli

include recent myocardial infarction (MI), an akinetic ventricular segment, dilated cardiomyopathy, a prosthetic heart valve, infective and nonbacterial thrombotic endocarditis, left heart myxoma, left atrial spontaneous contrast echo, and atrial septal aneurysm.

(2) A patent foramen ovale (PFO) or atrial septal defect predisposes patients to paradoxical emboli, especially if there is a documented venous thrombosis. Patients younger than 55 years of age with a PFO and an associated atrial septal aneurysm may be at particularly high risk for embolic stroke.

(3) Other cardiac conditions such as mitral valve prolapse or a hypokinetic ventricular segment are rarely associated with a cardiogenic embolus.

o b. Diagnosis. The diagnosis is most certain if there is an abrupt onset of neurologic dysfunction, an underlying cardiac condition known to predispose to emboli, strokes in multiple vascular territories, hemorrhagic arterial infarction, systemic emboli, an absence of concurrent conditions known to cause stroke, and angiography demonstrating (potentially transient) vessel occlusions in the absence of an intrinsic vasculopathy. Patients who have a cardiac embolism rarely present with all of these conditions.

o c. Therapy. An ischemic infarction caused by a cardiogenic embolus may develop into a hemorrhagic infarction, especially if reperfusion

Page 35: Neurology

occurs or the infarction is large. Therefore, care must be taken to lessen the risk of parenchymal hemorrhage in acutely ill patients while simultaneously taking measures to protect them from another embolic stroke.

(1) If the patient has had a relatively small ischemic embolic infarction, a CT scan should be obtained. If no blood is present, a continuous infusion of heparin is administered. A partial thromboplastin time (PTT) greater than twice the control value should be avoided to minimize the risk of hemorrhagic conversion of the ischemic infarction and clinical worsening.

(2) If the patient has had a large ischemic embolic infarction, anticoagulation therapy should be withheld for 5–7 days. If the patient is recovering and a subsequent CT scan reveals no blood, heparin may be administered.

(3) Subsequent treatment with oral anticoagulants depends on the underlying disease process and the patient's general condition.

4. Large artery disease o a. Aortic arch atheromas. Or thrombi. These thrombi, which are

visualized by transesophageal echocardiography, can embolize to the cerebral circulation and cause stroke. Optimal management is yet to be defined but usually involves antiplatelet or anticoagulation therapy, with the latter being used for mobile, pedunculated thrombi.

o b. Asymptomatic cervical bruit and carotid stenosis. The combination of an internal carotid artery bruit and atherosclerosis at the internal carotid artery origin is a marker for coronary artery disease as well as cerebrovascular disease. Approximately 2% of these patients will have an ischemic stroke each year. Patients with a hemodynamically significant or progressive stenosis are at increased risk for cerebral infarction.

(1) Etiology. A midcervical bruit can be caused by a hyperdynamic circulation (e.g., as in anemia, pregnancy, and thyrotoxicosis), an external carotid artery stenosis, an internal carotid artery stenosis, or a venous hum.

(2) Diagnosis. Noninvasive vascular testing (see I C 3 d) is helpful in determining whether the bruit originates from atherosclerotic internal carotid artery disease and whether the lesion is hemodynamically significant. Angiography is usually used if the patient is a surgical candidate.

(3) Therapy (a) Control of risk factors for atherosclerosis (see

Chapter 1 III A 2). Antiplatelet therapy (aspirin, clopidogrel, or aspirin combined with slow-release dipyridamole) is often prescribed, and a careful coronary artery evaluation is warranted.

P.539

Page 36: Neurology

(b) Medical treatment. Patients with less than a 50% internal carotid artery stenosis should be managed medically.

(c) Carotid endarterectomy (i) Patients with a 50%–99% stenosis who

undergo carotid endarterectomy have less of a risk of ipsilateral stroke than patients managed medically if surgery can be achieved with less than a 3% risk of complications.

(ii) Prophylactic carotid endarterectomy for unilateral, slightly or moderately stenotic, asymptomatic internal carotid artery disease before major cardiac or vascular surgery is usually inadvisable.

o c. Ischemic infarction. An understanding of neuroanatomy is essential to localize the compromised area of the brain and to correlate this information with a likely site of vascular disease.

(1) Etiology (a) Large artery occlusive disease can cause ischemic

infarction, either by being a source of artery-to-artery emboli or by causing hypoperfusion distal to a hemodynamically significant vascular stenosis. Large artery disease can involve the extracranial or intracranial portions of the cerebrovascular circulation.

(b) Vascular conditions other than atherosclerosis (e.g., arterial dissection, arteritis, Takayasu's syndrome, fibromuscular dysplasia, and radiation-induced vasculopathy) should be considered.

(2) Diagnosis. Diagnostic studies are pursued to define the vascular anatomy. CTA, MRA, carotid duplex, TCD, and conventional angiography can be used. The scope of testing is determined, in part, by the clinical condition of the patient and whether carotid endarterectomy or antithrombotic therapy is primary therapeutic considerations. The more accurate the definition of the underlying causes of the infarction, the more precise the determination of prognosis and treatment.

(3) Therapy (a) Extracranial carotid artery disease. If the patient has

sustained a minor infarction with a functional recovery and has a 50%–99% atherosclerotic stenosis of the ipsilateral origin of the internal carotid artery, carotid endarterectomy is superior to medical therapy for prevention of subsequent stroke. If the stenosis is less than 50%, therapy with antiplatelet drugs is appropriate.

(b) Intracranial large artery disease (i) If the infarction is caused by a severe stenosis

or occlusion of a large intracranial artery, recent evidence suggests that aspirin therapy is safer than, and equally effective as, anticoagulation therapy.

Page 37: Neurology

(ii) Some physicians use anticoagulation agents if the patient has new symptoms.

5. Small artery disease. The infarctions resulting from small artery disease typically are deep in the hemispheres or the pontomesencephalic region and are known as lacunae. The lesions are less than or equal to 15 mm in diameter.

o a. Etiology. The underlying vascular lesion is usually hypertension-associated lipohyalinosis. Diabetes mellitus is also associated with lacunar infarcts. A small atheromatous plaque blocking the ostium of an arteriole may, on occasion, be the cause of some occlusions, as may infectious or sterile inflammation, cardiogenic emboli, and large artery occlusive disease.

o b. Diagnosis (1) Ischemic events from small artery disease cause stereotypic

syndromes such as pure motor or sensory stroke, sensorimotor stroke, clumsy hand–dysarthria syndrome, and ataxic hemiparesis.

(2) Patients should be screened for the possibility of cardiogenic emboli, large artery occlusive disease, and hematologic and inflammatory disorders (see VIII B) if they are not hypertensive or if their history, examination, or routine diagnostic tests suggest a cause other than hypertension.

o c. Therapy is directed at controlling hypertension. If another condition is defined as the cause of the infarction, appropriate intervention should be pursued. Antiplatelet therapy may be administered to decrease the likelihood of subsequent ischemic stroke.

6. Hematologic and systemic conditions. These disturbances are associated with ischemic infarction.

P.540

o a. Associated conditions. Sickle cell disease, hyperviscosity associated with polycythemia and paraproteinemias, and hypercoagulability are conditions associated with ischemic stroke.

o b. Etiology. Hypercoagulability is associated with antiphospholipid antibodies (including anticardiolipins and the lupus anticoagulant syndrome), hyperhomocysteinemia, deficiency of proteins C and S, activated protein C resistance, antithrombin III deficiency, malignancy, nephrotic syndrome, and pregnancy, as well as several other conditions.

7. The young ischemic stroke patient. Patients younger than 45 years of age who present with stroke are often diagnostic challenges. The potential etiologies are vast and include, but are not limited to, the following conditions:

o a. Drug (especially cocaine) and alcohol abuseo b. Hypercoagulable stateso c. Cardiogenic embolio d. Migraineo e. Vasculitis and other rare arterial lesions

Page 38: Neurology

o f. CNS infection, including HIV-associated conditionso g. Cancero h. Disorders of homocysteine metabolismo i. Familial conditions [e.g., neurofibromatosis (NF), von Hippel-

Lindau disease)]o j. Pregnancy and the postpartum state

8. The deteriorating ischemic stroke patient o a. Pathophysiology.

(1) The patient's condition may deteriorate as a result of progressive occlusion of arteries from clot propagation or artery-to-artery emboli or because of subsequent cardiogenic emboli. Deterioration also may result from excessive lowering of blood pressure or inadequate anticoagulation therapy.

(2) Hemorrhagic infarction or a parenchymal hematoma can occur spontaneously or as a result of thrombolytic or anticoagulation therapy.

(3) If heparin is being used as treatment, the possibility of heparin-induced thrombosis (with associated thrombocytopenia) should be considered.

(4) Cerebral edema can develop, causing shifting of brain structures and an increase in the ICP.

o b. Approach to the patient. Ideally, continuing efforts should be made to define the pathophysiology of the stroke.

(1) The blood pressure and degree of hydration should be checked.

(2) A CT scan should be obtained to define mass effect and hemorrhagic complications.

(3) A hematocrit, platelet count, and clotting profile should be obtained as appropriate.

o c. Therapy. Patients should be confined to bed, and extremes of blood pressure should be avoided.

(1) Appropriate hydration should be maintained with normal saline, and increases in ICP should be treated as necessary.

(2) Consideration can be given to novel, interventional neuroradiologic techniques such as intra-arterial thrombolysis and angioplasty, depending on the degree of neurologic impairment, the time course of the illness, and the availability of resources.

C. Hemorrhagic disorders

1. Subarachnoid hemorrhage (SAH) o a. Etiology. The most common causes of SAH are trauma and ruptured

berry aneurysms. (1) Other causes include Coagulopathies, mycotic aneurysm,

arteriovenous malformation, vasculitis, and sympathomimetic drugs.

(2) Aneurysms may be familial and are associated with polycystic kidney disease, coarctation of the aorta, fibromuscular dysplasia, moyamoya disease, polyarteritis

Page 39: Neurology

nodosa, pseudoxanthoma elasticum, and Marfan and Ehlers-Danlos syndromes.

o b. Diagnosis (1) Clinical signs. Patients who have a ruptured berry aneurysm

complain of “the worst headache of my life,” but examination may not reveal many objective findings. Other

P.541

Patients can present with meningismus, altered states of arousal, and focal neurologic findings. Partial third nerve palsy with pupillary dilation is suggestive of a posterior communicating artery aneurysm.

(2) Diagnostic studies. A CT scan reveals the presence of subarachnoid blood in most patients (Figure 11-3). If the index of suspicion for an SAH is high, but a CT scan is unrevealing, an LP provides the proper diagnosis. Conventional interventional angiography is necessary to characterize the aneurysm.

o c. Therapy. Treatment of aneurysmal SAH is aimed at controlling complications, which include re-bleeding, vasospasm leading to delayed ischemic stroke, hyponatremia, acute or chronic hydrocephalus, intraparenchymal and intraventricular hematoma, and cardiac arrhythmias. Unfortunately, the mortality rate from aneurysmal rupture approaches 50%, although recent studies suggest improved outcomes. Interventional neuroradiologic techniques are playing a larger role in the management of aneurysms.

(1) Re-bleeding most frequently occurs in the first 48 hours after aneurysmal rupture. Early intervention to isolate the aneurysm should be performed when possible to eliminate the threat of rebleeding. Surgical “clipping” or endovascular therapy, which involves placing thrombogenic “coils” in the aneurysm, can be used to isolate the aneurysm.

(2) The course of vasospasm can be followed with TCD and CTA studies, and treated accordingly.

(a) The development of ischemic stroke from vasospasm is less likely when nimodipine which is a calcium channel blocker that may decrease small artery vasoconstriction or provide neuronal protection from ischemia, is administered.

(b) Prophylaxis against symptomatic vasospasm also includes maintaining patients in a euvolemic state and avoiding hypotension.

(c) If the aneurysm has been isolated from the circulatory system, symptomatic vasospasm may be treated with hypervolemic therapy, coupled with a moderate increase in blood pressure.

Page 40: Neurology

(d) Refractory vasospasm may be amenable to angioplasty or selective intra-arterial papaverine or calcium antagonist infusion.

View Figure

FIGURE 11-3 A nonenhanced computed tomography (CT) scan demonstrating a subarachnoid hemorrhage (SAH). Note the blood in the basal cisterns and the sylvian fissures.

o P.542o

(3) Symptomatic hydrocephalus can be treated with a ventricular drain, repeated LPs, or ventriculoperitoneal shunting, as dictated by clinical circumstances.

2. Intraparenchymal hematoma o a. Diagnostic considerations. Chronic hypertension is often, but not

invariably, associated with hemorrhage in the region of the putamen, thalamus, cerebellum, and pons.

(1) An intraparenchymal hemorrhage that is not accompanied by a history of hypertension should prompt a search for an underlying cause of bleeding (e.g., coagulopathy, aneurysm, arteriovenous malformation, or tumor), particularly if the hemorrhage is not located in a region of the brain typically associated with hypertensive bleeding.

(2) In elderly patients, lobar hematomas, especially if multiple, may be indicative of amyloid angiopathy. The APOE4 allele may be a risk factor for amyloid angiopathy and amyloid angiopathy-related hemorrhage.

(3) Coagulopathies (especially when induced by thrombolytic therapy) and the use of drugs such as cocaine and sympathomimetics are associated with intraparenchymal hematoma.

o b. Diagnosis (1) Clinical signs

Page 41: Neurology

(a) A large putaminal hematoma causes impaired consciousness, contralateral hemiparesis and sensory loss, and gaze preference to the side of the hemorrhage.

(b) Thalamic hemorrhage can lead to impaired consciousness; contralateral weakness and sensory loss; diminished vertical gaze; and small, poorly reactive pupils (i.e., Parinaud's syndrome).

(c) Patients with cerebellar hemorrhage may present with dizziness, impaired gait and stance and limb ataxia. If the hematoma is large, impaired consciousness, cranial nerve palsies (including eye movement abnormalities), and weakness can develop.

(d) The classic signs of pontine hemorrhage include coma, pinpoint reactive pupils, impaired lateral ocular motility, and quadriplegia with decerebrate posturing. Small pontine hemorrhages cause more restricted pontine syndromes.

(2) Diagnostic studies. A CT scan is central to diagnosis. A coagulation profile and drug toxicology screen should be performed. Angiography may be appropriate in normotensive patients and in those with hemorrhage at atypical sites.

o c. Therapy (1) Taking measures to lower the elevated ICP associated with

parenchymal hematoma can improve outcome. (2) With cerebellar hematomas, surgical resection is a

lifesaving measure. Surgical resection of hematomas at other sites is receiving increasing attention.

(3) A coagulopathy should be treated appropriately. 3. Arteriovenous malformation. Headaches, seizures, and intraparenchymal or,

occasionally, SAHs may result. Therapeutic intervention may incorporate multiple modalities, including surgery, interventional radiology with embolization, and stereotactic radiosurgery.

IX. Seizures and EpilepsyA seizure involves a sudden abnormality of brain electrical activity. Manifestations of a seizure can include impairment or loss of consciousness and sensory, motor, or behavioral abnormalities. The term epilepsy describes a syndrome characterized by recurrent seizures.A. ClassificationOptimal management of seizure patients depends on proper classification of the seizure type. Seizures can be categorized as generalized or partial (focal).

1. Generalized seizures are characterized by a sudden loss of consciousness.o a. Generalized convulsive seizures. Consist of tonic, clonic, or tonic–

clonic (grand mal) motor activity. Generalized convulsions can result from a focal seizure disorder that has spread, involving the entire brain. Postictal obtundation and confusion commonly last minutes, and occasionally, hours. The EEG often shows generalized spikes or spikes and associated slow waves.

Page 42: Neurology

o b. Absence seizures are characterized by a brief, staring spells without a post-ictal state. Typically, the EEG demonstrates generalized 3-Hz spikes and associated slow waves.

P.543

2. Simple partial (focal) seizures are not accompanied by an impairment of consciousness. Generalized motor seizures may develop secondarily.

o a. There may be isolated clonic or tonic activity of a limb or transient altered sensory perceptions.

o b. The seizure activity may spread over one side of the body in a Jacksonian march (e.g., the convulsive activity can start in the face, move to the ipsilateral arm and then to the leg, and may evolve into a generalized seizure).

o c. The EEG may show a focal rhythmic discharge at the onset of a simple partial seizure, but occasionally, no ictal activity is detected. Interictally, focal spikes with associated slow waves are frequently present.

3. Complex partial (focal) seizures are often characterized by an aura followed by impaired awareness. The EEG often shows interictal spikes or spikes with associated slow waves in the temporal or frontotemporal areas and ictal focal rhythmic discharges. Generalized motor seizures may develop secondarily.

o a. The aura may involve hallucinations (e.g., olfactory, visual, auditory, or gustatory) and complex illusions (e.g., of having experienced a new event or of never having experienced a commonplace event). However, patients frequently do not recall their aura.

o b. Nausea or vomiting, focal sensory perceptions, and focal tonic or clonic activity may accompany a complex seizure.

o c. After the aura, there may be an episode of impaired consciousness, lasting seconds to several minutes, during which time automatisms may be observed. Return to baseline cognitive abilities can take several minutes.

4. Status epilepticus is defined as an episode of repeated or ongoing seizure activity with impaired arousal lasting at least 30 minutes. Status epilepticus may involve both convulsive (generalized tonic–clonic activity) and nonconvulsive (absence or complex partial) seizures.

o a. Patients experiencing convulsive seizures are at risk for hypoxia, aspiration, acidosis, hypotension, hyperthermia, myoglobinuria, hypoglycemia, and multiple physical injuries.

o b. Patients experiencing nonconvulsive seizures can appear delirious. A fluctuating sensorium and subtle automatisms or myoclonic jerks are clues to the diagnosis, and the EEG is confirmatory.

B. EtiologyTable 11-6 outlines some of the many causes of seizures. Several seizure types have a defined genetic basis: autosomal dominant frontal lobe epilepsy (neuronal nicotinic

Page 43: Neurology

acetylcholine receptor mutation) and autosomal dominant temporal lobe epilepsy with auditory features.TABLE 11-6 Selected Causes of Seizures IdiopathicGenetic predispositionMesial temporal sclerosisMetabolic abnormalities   Hyponatremia   Hypo- or hyperglycemia   Hypocalcemia   Hypomagnesemia   UremiaVascular disease and stroke   Infarction, especially cortical   Vascular malformation   VasculitisInflammatory causes   Systemic lupus erythematosus (SLE)Neoplasia   Metastatic and primary brain tumorsInfection   Meningitis, abscess, and encephalitisDegenerative diseases   Alzheimer's diseaseTraumaEclampsiaDrugs   Theophylline   Lidocaine   CocaineDrug and substance withdrawal   Anticonvulsant medications   Benzodiazepines   AlcoholPsychogenic causes

P.544

C. Diagnosis

1. Patient history and physical examination can aid in the determination of whether a seizure or some other transient event was responsible for the patient's symptoms.

o a. A family history of epilepsy or a history of febrile convulsions is relevant.

o b. An accurate description of the event by an observer is helpful in defining the problem.

o c. Urinary incontinence, back pain (from a vertebral compression fracture), myalgias, and oral lacerations are clues to proper diagnosis.

Page 44: Neurology

o d. Fever, fatigue, stress, alcohol withdrawal, medications, and menses can provoke seizures.

2. Differential diagnosis. Other conditions that may produce sudden loss of consciousness are discussed in II A. Psychogenic seizures should be considered if patients exhibit nonstereotypic events, have an unexpected resistance to antiepileptic drugs, or have a psychiatric disorder.

3. Diagnostic studies o a. The EEG is central to the evaluation of seizure patients. The best

technique for fully characterizing a seizure disorder is continuous video EEG monitoring, but this is not usually used as an initial diagnostic test. A normal EEG does not exclude the diagnosis of epilepsy.

o b. An MRI scan is the most useful modality for detecting lesions that may cause seizures.

o c. SPECT and positron emission tomography (PET) can provide additional information to help localize a seizure focus.

o d. Laboratory studies are indicated to evaluate potential metabolic or toxic causes, including glucose, sodium, calcium, blood urea nitrogen (BUN), creatinine (Cr), liver fucntion tests (LFTs), and toxin screen.

D. Therapy

1. General considerations. If a seizure is the suspected diagnosis, decisions about therapy depend on the underlying cause.

o a. Correction of hyponatremia, hypoglycemia, or drug intoxication may be all that is necessary.

o b. Patients with a neurologic condition known to be associated with recurrent seizures often require medication.

o c. Anticonvulsant therapy is often not initiated in patients with a single, unprovoked convulsion; a normal neurologic examination; and a normal brain imaging study and EEG unless they experience a second seizure.

2. Medical therapy o a. General principles. An attempt is usually made to prevent

subsequent seizures by using a single agent, to limit toxic effects. The drug should be administered in progressive doses until seizure control has been achieved or until drug toxicity occurs. Only if monotherapy fails should a second drug be added. If control is then obtained, the first agent might be carefully withdrawn.

o b. Specific agents. The choice of medication should be based on the seizure type, bearing in mind possible contraindications and side effects.

(1) Typically, generalized convulsive, simple partial, and complex partial seizures are treated with carbamazepine, phenytoin, Valproic acid, topiramate, levetiracetam, Lamotrigine, or zonisamide.

(2) Valproic acid or Ethosuximide is used for generalized nonconvulsive spells (absence seizures).

Page 45: Neurology

(3) Valproic acid is particularly effective for controlling juvenile myoclonic epilepsy (a disorder characterized by myoclonic seizures).

(4) Lamotrigine, gabapentin, tiagabine, oxcarbazepine, and topiramate are adjunctive medications for the treatment of patients with refractory partial seizures.

3. Surgical therapy. Patients refractory to medical control of seizures may be candidates for surgery to control the epilepsy. Temporal lobe resection, ablation of a cortical seizure focus, and corpus callosum sectioning are able to reduce seizure frequency in some patients who meet specific criteria. Vagus nerve stimulation may help control seizures.

4. Control of status epilepticus. Generalized convulsive status epilepticus is a life-threatening condition; therefore, management of convulsive status epilepticus requires making certain that the

P.545

Airway is unobstructed and maintaining adequate oxygenation, blood pressure, and hydration. Definition of the underlying problem is essential.

o a. Glucose and thiamine should be administered after blood samples for glucose, electrolytes, renal function, anticonvulsant drug levels, and toxicology have been obtained.

o b. Often, intravenous diazepam or lorazepam is given to stop the convulsions. Lorazepam carries less of a risk of respiratory depression or arrest and remains effective for longer periods.

o c. Administration of a benzodiazepine is followed by administration of phenytoin, fosphenytoin, or Phenobarbital.

o d. If convulsions continue after loading doses of phenytoin or fosphenytoin or Phenobarbital have been administered, intravenous midazolam, propofol, or pentobarbital can be given in a carefully supervised setting, with continuous EEG monitoring, until the seizure discharges are eliminated from the EEG.

o e. Diminished cardiac output, bradycardia, and hypotension often limit the dose of intravenous anticonvulsants. Fluid resuscitation and vasopressors may be used.

5. Psychosocial issues. The following issues are important to consider when managing epilepsy.

o a. Patients may be depressed, have behavioral disturbances, and often require vocational support services. Frequently, community support services are available.

o b. Review of the patient's driving status and work and play environment are necessary.

o c. Family and friends need to be counseled as to how to manage a convulsion.

o d. Women of childbearing age should be counseled about issues relating to pregnancy.

Page 46: Neurology

o e. Pregnant women should be maintained on monotherapy at the lowest effective dose, given that seizures may harm the fetus, and the risk of birth defects increases with polypharmacy.

X. Movement DisordersA. Parkinson's disease

1. Pathogenesis o a. Parkinson's disease is characterized by a degeneration of cells in the

substantia nigra, which causes a deficiency of dopamine (a neurotransmitter) in the CNS, leading to a series of changes in motor control pathways. The mechanism behind the degeneration of these cells is unknown, although hypotheses center about free radical damage and impaired mitochondrial oxidative function.

o b and c. Genetics. Major insights into the genetics of Parkinson's disease are emerging. The SNCA gene codes for α-synuclein, the PRKN gene codes for parkin, and the UCHL1 gene codes for a ubiquitin hydroxylase. Several other chromosome loci have been associated with Parkinson's disease.

o c. A common feature emerging from the genetics studies is that the Pathophysiology of Parkinson's disease, as well as that of other neurodegenerative diseases, may involve abnormalities in pathways associated with ubiquitin-associated protein degradation.

2. Diagnosis o a. Clinical symptoms and signs.

(1) Parkinson's disease patients often complain of “slowing down”; they have trouble dressing, arising from a seated position, climbing or descending stairs, writing, and turning over in bed.

(2) On examination, rigidity and akinesia or bradykinesia are present and a 3-Hz resting tremor and postural instability are often evident. Signs are often asymmetrical early in the disease.

(3) Cognitive impairment develops in more than 50% of patients over time.

o b. Differential diagnosis. The diagnosis is a clinical determination, although, at times, testing to exclude other entities presenting as Parkinsonism is indicated.

(1) Sometimes serial observations are necessary to determine whether the parkinsonian state is the harbinger of another neurologic illness; this is of special concern in patients who do not have the characteristic “pill-rolling” or resting tremor of Parkinson's disease or fail to respond to levodopa therapy.

(2) Conditions that may produce parkinsonian symptoms similar to those found in patients with Parkinson's disease include NPH, multiple strokes, hypothyroidism, drug effects [e.g., neuroleptics (dopamine-blocking agents), metoclopramide, diltiazem, and reserpine], Wilson's disease, anoxic encephalopathy, and intoxication [e.g., by carbon monoxide, manganese, or n-methyl-4-phenyl-1,2,3,6-tetrahydropyridine (MPTP)].

Page 47: Neurology

(3) Rare neurologic disorders that may have parkinsonian

features are summarized in Online Table 11-7.

o P.546o

ONLINE TABLE 11-7 Key Features of Selected Conditions Causing Parkinsonism

DisorderDistinguishing Clinical CharacteristicsProgressive supranuclear palsy (“tauopathy”)

Impaired vertical gaze; early axial rigidity with postural instability

Multiple systems atrophy (Shy-Dragger syndrome)

Autonomic insufficiency; cerebellar dysfunction; upper and lower motor neuron dysfunction

Diffuse Lewy body disease (“Dementia with Lewy bodies”)

Dementia early in illness, fluctuating cognition, visual hallucinations

Cortical-basal ganglionic degeneration (“tauopathy”)

Asymmetric findings on examination of sensory loss and apraxia; unilateral rigidity; unilateral stimulus-sensitive myoclonus; dystonia

Frontotemporal lobar degeneration (“tauopathy”)

Apathy, disinhibition, anomia, effortful speech

Huntington's diseaseParkinsonian features prominent in young patients; family history, choreoathetosis

Olivopontocerebellar atrophy*

Cerebellar dysfunction; autonomic dysfunction

Basal ganglia calcificationCalcification visible on computed tomography (CT) scan

NeuroacanthocytosisAcanthocytes in wet peripheral blood smear*May be the same as multiple systems atrophy.

3. Therapy. Parkinson's disease is a progressive disease. Therefore, management protocols vary depending on the patient's symptoms and the extent of functional impairment.

o a. Early therapy. Several medications are available to treat Parkinson's disease.

(1) Carbidopa/levodopa combinations are the mainstay of treatment for Parkinson's disease. This treatment should begin when the disease impairs the patient's functional status. A sustained release formulation of carbidopa/levodopa is available and provides a more uniform clinical response than conventional dosing.

Page 48: Neurology

(a) Levodopa is converted to dopamine by the presynaptic neuron and therefore increases the amount of neurotransmitter available to the postsynaptic dopamine receptor.

(b) Carbidopa blocks systemic conversion of levodopa to dopamine, thereby decreasing the undesirable systemic effects of levodopa.

(2) Dopamine agonists are increasingly being used at initial therapy, especially in younger patients. They may have some neuroprotective benefits. Drugs include pramipexole, ropinirole, pergolide, and bromocriptine.

(3) Anticholinergics, which improve the cholinergic–dopaminergic balance in the basal ganglia, are particularly helpful in treating tremor. However, they may contribute to cognitive impairment.

(4) Amantadine, which increases the availability of dopamine to the postsynaptic neuron, can be effective early in the course of the disease or as an adjunctive therapy later in the disease course to help “smooth out” motor function.

o b. Advanced therapy. In the later stages of the disease, therapy is directed at optimizing the patient's functional status and avoiding adverse effects of medication.

(1) Dopamine agonists. If the therapeutic response to carbidopa/levodopa therapy is inadequate, or if the patient cannot tolerate the medication, pramipexole, ropinirole, pergolide, or bromocriptine may be administered.

(a) These drugs are direct postsynaptic dopamine-receptor agonists.

(b) A combination of carbidopa/levodopa and a dopamine agonist seems to be particularly effective and is often well tolerated. Dopamine agonists help decrease motor fluctuations when used in conjunction with carbidopa/levodopa.

(2) Management with disease progression. Management of Parkinson's disease becomes increasingly difficult as the disease progresses. “Wearing off” effects, dyskinesias, and wide, random swings in patient mobility (“on–off” phenomena) develop. A sustained-release form of carbidopa/levodopa (alone or in combination with a dopamine agonist) can be used. Catechol O-methyltransferase inhibitors, which increase the synaptic availability of levodopa by blocking its degradation, also can be used to manage unstable patients.

o c. Ancillary therapy (1) Other therapeutic maneuvers include the strategic reduction

of medication if patients are experiencing dyskinesia and judicious use of psychotropic agents to treat the various untoward behavioral consequences of Parkinson's disease (e.g., insomnia, hallucinations, agitation).

(2) Dietary manipulations that redistribute or limit protein intake during the day may improve the efficacy of levodopa.

Page 49: Neurology

(3) Physical therapy and an exercise program help optimize mobility.

(4) Pallidotomy and deep brain stimulation offer new therapeutic options for refractory Parkinson's disease patients.

(5) The role of surgical implants of dopamine-containing cells for the treatment of Parkinson's disease remains experimental.

B. Hyperkinetic disorders

1. Tremor o a. Benign essential tremor. Is characterized by a posture-related 5–9-

Hz oscillation of the hands and forearms that impairs performance of fine motor tasks.

(1) This type of tremor is often familial and may be accompanied by titubation (head tremor).

(2) Consumption of alcohol may temporarily suppress the tremor; stress, caffeine, or sleep deprivation may exacerbate the condition.

(3) β-Adrenergic blocking agents and primidone are effective treatments.

P.547

o b. An action (kinetic) tremor is evident when patients move their arms; there may be a relatively mild accompanying postural and intention component. Treatment with clonazepam may be useful.

2. Chorea describes rapid, “dance-like” distal limb and facial movements. Causes include hyperthyroidism, drugs (e.g., birth control pills and levodopa), Sydenham's chorea, pregnancy, SLE, antiphospholipid syndrome, stroke, porphyria, Wilson's disease, Lyme disease, Huntington's disease, and neuroacanthocytosis.

3. Athetosis generally refers to involuntary, slow, writhing, “snake-like” limb movements. Causes include Wilson's disease, Huntington's disease, anoxic encephalopathy, trauma, birth control pills, and several rare hereditary disorders.

4. Dystonia describes slow, writhing, sustained and involuntary contractions of the proximal limb, trunk, and neck musculature. Dystonia is associated with Wilson's disease, Parkinson's disease, Huntington's disease, trauma, neuronal storage disorders, encephalitis, drugs (e.g., neuroleptics, levodopa), and other rare hereditary conditions.

o a and b. Genetics. The TOR1A gene for autosomal dominant generalized torsion dystonia codes for torsin A, an ATP-binding and heat shock protein. Patients may respond to high doses of trihexyphenidyl.

o b. Some patients with autosomal dominant idiopathic dystonia are particularly responsive to carbidopa/levodopa therapy. This condition is associated with the DYT5 gene on chromosome 14 that codes for

Page 50: Neurology

guanine triphosphate (GTP) cyclohydrolase I; a deficiency of this enzyme causes loss of dopamine synthesis.

o c. Focal dystonias such as writer's cramp, blepharospasm, spastic dysphonia, and torticollis can occur. Treatment with local botulinum toxin infiltration can be beneficial.

o d. Some patients with dystonia may exhibit chorea and athetosis. 5. Hemiballismus describes wild, flinging, principally proximal movements of

the arms or legs. It is often caused by an infarct in the subthalamic nucleus. Haloperidol can decrease the involuntary movements.

6. Blepharospasm can occur in isolation or as part of a more widespread disorder such as Parkinson's disease, stroke, or Meige's syndrome (orofacial dystonia). Blepharospasm can be of such severity as to cause functional blindness.

o a. Many drugs have been tried in an attempt to control the problem with modest effect.

o b. Infiltration of botulinum toxin about the eyes can provide relief by decreasing neuromuscular transmissions.

7. Neuroleptic-associated movement disorders represent a spectrum of disorders related to the acute or chronic administration of neuroleptic medications (although for selected conditions, other drugs are implicated as well).

o a. Acute dystonia. Typically occurs shortly after the first few doses of a neuroleptic agent.

(1) Clinical signs include uncontrollable face, neck, tongue, and eye muscle (oculogyric crisis) spasms.

(2) Therapy consists of the administration of anticholinergics or Diphenhydramine.

o b. Parkinsonism can develop with neuroleptic use. Therapy consists of decreasing the dose of the neuroleptic agent; changing to another neuroleptic drug; administering an anticholinergic agent; or using amantadine, a carbidopa/levodopa preparation, or an “atypical” neuroleptic such as clozapine, risperidone, or olanzapine.

o c. Tardive dyskinesia is an almost constant writhing movement of the tongue and oromandibular area, which may be accompanied by blepharospasm, respiratory grunts, choreo-athetosis, and truncal hyperactivity. Ill-fitting dentures or an edentulous state can cause mouthing movements that are mistaken for tardive dyskinesia.

(1) Tardive dyskinesia is usually an adverse side effect of long-term use of neuroleptic agents; occasionally other drugs such as amphetamines, antihistamines, and carbamazepine are causally implicated.

(2) If a medication is implicated as the cause of the problem, the offending agent should be discontinued if possible. Clonazepam, reserpine, tetrabenazine, and other drugs have been used to treat tardive dyskinesia with variable success. Use of an “atypical” neuroleptic such as clozapine is another option.

o d. Other neuroleptic-associated movement disorders include tardive dystonia, akathisia (motor restlessness), and the rabbit syndrome (rhythmic lip movements).

Page 51: Neurology

o e. Neuroleptic malignant syndrome (NMS) is an idiosyncratic reaction to neuroleptic agents. It can also occur in Parkinson's disease patients after the abrupt discontinuation of antiparkinsonian medications. Dopamine receptor blockade is thought to be the cause of NMS.

P.548

(1) Clinical signs include altered mentation, high fever, rigidity, autonomic instability, high creatine kinase (CK) levels, and myoglobinuria.

(2) Therapy for this potentially lethal condition includes hydration, cooling blankets, antipyretics, dantrolene, and levodopa/carbidopa preparations or bromocriptine, although the use of antiparkinsonian drugs is controversial.

8. Meige's syndrome (orofacial dystonia) is an idiopathic condition that has features of dystonia and tardive dyskinesia, particularly blepharospasm.

o a. This diagnosis cannot be made if the patient has recently taken neuroleptic agents or other drugs implicated as a cause of tardive dyskinesia.

o b. No single medication is consistently effective in treating this condition; clonazepam is a reasonable first-line agent. Botulinum toxin infiltration can be used to treat the blepharospasm.

9. Hemifacial spasm describes lightning-quick spasms of muscles innervated by the facial nerve.

o a. The condition is most often caused by irritation of the facial nerve by a vascular “loop.”

o b. Rarely, it occurs after facial nerve paralysis or is associated with tumors, MS, or other irritative processes.

o c. Botulinum toxin infiltration, clonazepam, and carbamazepine have been used to treat hemifacial spasm; microsurgical decompression may be successful.

10. Tics are brief, stereotypical, involuntary movements, sounds, or sensations that occur within the context of normal neurologic function.

o a. Types of tics. (1) Simple motor tics are isolated movements such as an

eyeblink, shoulder shrug, or facial grimace. Complex motor tics include touching, smelling, and jumping.

(2) Simple phonic tics include throat clearing, sniffling, and grunting. Complex phonic tics include the repetition of words and coprolalia.

(3) Sensory tics often accompany motor and phonic tics and are characterized by focal sensations of pressure, tickle, warmth, or cold.

o b. Tourette's syndrome (1) Characteristics

(a) Multiple motor and one or more phonic tics (that has been present at some time over the course of the illness)

Page 52: Neurology

(b) Tics that occur many times a day, nearly every day, for more than 1 year

(c) Tics that change over time in their anatomic location, number, frequency, complexity, type, and severity

(d) Onset of illness before age 21 (e) Absence of other conditions that can cause similar,

but isolated, symptoms (e.g., neuroleptic drug effects, seizures, and chorea)

(2) Etiology. Tourette's syndrome is thought to be inherited in a polygenic manner. Striatal dopamine receptor supersensitivity may be responsible for the clinical manifestations.

(3) Associated behavioral disturbances. Obsessive–compulsive disorder and attention deficit hyperactivity disorder may occur.

(4) Therapy. Treatment consists of education and counseling of the patient, family, and other appropriate parties. Clonidine, pimozide, or haloperidol can be used to manage disabling tics. Behavioral disturbances should respond to appropriate psychoactive medications.

XI. Demyelinating DiseasesA. Multiple sclerosis (MS)Multiple sclerosis (MS) is characterized by multiple foci of CNS demyelination.(thought to be a cell mediated autoimmune disease associated with immune activity directing against CNS antigens, principally myelin) Patients either experience clinical remissions (often followed by relapses) or chronic, progressive symptoms.

1. Pathophysiology . Although the cause & etiology of MS remains unknown (there is a genetic component with an increased relative risk /20-40% in siblings compared to the general population. However, as the concordance rate in monozygotic twins is only 25%, there appears to be environmental component as well.), a predominant theory contends that MS is an immunologic disorder associated with CNS immunoglobulin production and alteration of T and B lymphocytes. The pathologic hallmark of MS is inflammation associated with areas of demyelination scattered about CNS white matter. Recent findings have also documented axonal disruption.

2. Diagnosis this typically needs the demonstration of brain or spinal cord lesions that are disseminated in time and anatomical location. A definitive diagnosis, therefore, is hard to make at the time of the first neurological episode. Diagnosis of primary progressive disease is often the hardest, as clear evidence for lesions disseminated at the time and anatomical space is often obscured by the progressive course of the disease. Brain MRI may be normal in this form of MS, although multiple lesions may be visible in the spinal cord.

o a. Clinical signs. 1. The diagnosis is most certain if neurologic problems occur

over an extended period and involve several white matter pathways. MRI findings can be used as evidence of disease dissemination over time and anatomic space.

2. Several patterns emerge that are suggestive of MS, including optic neuritis (is a common presentation in which color vision

Page 53: Neurology

is affected early and a residual abnormality may persist after recovery, it causes a painful visual loss. Isolated optic neuritis gives 40%-60% chance of developing subsequent MS) (a sign of which is the Marcus-Gunn pupil or afferent pupillary defect) and internuclear ophthalmoplegia, either in isolation or in association with corticospinal tract or cerebellar signs. Long-term follow-up indicates that 74% of women and 34% of men who present with isolated optic neuritis ultimately develop MS.

3. Neurologic signs that can be localized to a single discrete area in the CNS such as the brainstem or craniocervical junction should suggest an alternate diagnosis such as a tumor or arteriovenous malformation.

Clinical consensus identifies 4 different subtypes of MS, which may reflect different immunological subtypes:

1) Relapsing-remitting disease: is the most common form of MS (80-85%). Short lasting acute attacks (4-8 weeks) are followed by remission and steady baseline state between relapses. The average number of relapses is around 0.8/year.

2) Secondary progressive disease: about 30-50% of pt with relapsing-remitting disease will subsequently show progressive deterioration with relapses becoming less prominent within about 10 yrs of MS disease onset.

3) Primary progressive disease: 10-15% show progressive deterioration from the onset without any superimposed relapses. Age of onset is typically later than for relapsing remitting disease.

4) Progressive relapsing disease: a small no. of pts with primary progressive disease also experience superimposed gradual disease progression.Good prognostic factors include:

Relapsing-remitting course. Female sex. Early age at onset. Presence of sensory symptoms.

o b. Differential diagnosis . There is no specific diagnostic marker for MS; the physician needs to exclude other conditions that can masquerade as MS. Among these disorders are somatization disorder, SLE, brainstem or spinal vascular malformation, Sjögren's syndrome, Lyme disease, HIV infection, vitamin B12 deficiency, brainstem neoplasm, vasculitis, sarcoidosis, and adrenomyeloleukodystrophy.

o c. Diagnostic studies 1. MRI (T2-weighted shows demyelinating plaques) is an

excellent technique for visualizing white matter lesions. Although its diagnostic specificity is poor, the contrast agent gadolinium-DTPA indicates areas of breakdown of the blood–

Page 54: Neurology

brain barrier. Serial MRI studies show white matter lesions that may come and go without clinical manifestations. (the presence of gadolinium enhancing lesions is the most predictive MRI parameter)

2. Examination of the CSF can indicate a sterile inflammation with a mild protein elevation; modest, predominantly mononuclear pleocytosis; an elevated IgG index; oligoclonal bands (but not in the serum, they indicate Intrathecal immunoglobulin synthesis. These are not specific to MS and other causes include: neurosarcoidosis, CNS lymphoma, SLE, neurosyphilis, SAH-rare-, subacute sclerosing panencephalitis (SSPE) a rare late complication of measles, Guillain-Barré syn. ); and increased myelin basic protein. Only occasionally do all of these abnormalities occur in a single patient.

3. (3) Visual, brain stem–auditory, and somatosensory EPs and central motor conduction studies can demonstrate clinically silent disruption of white matter tracts.(delayed VEPs)

3. Therapy . There is no cure for MS.o a. Corticosteroid therapy. May hasten maximal recovery from an

acute exacerbation. If optic neuritis is treated, high doses of intravenous corticosteroids are preferable to lower, oral doses.(NICE guidelines suggest that pts suffering acute attack should be treated with methyl-prednisolone, either orally (500mg-2g daily) or IV(500mg-1g daily) for 3-5 days. Steroids have no incidence of relapses, and are not useful other than for treatment of acute attacks).

o b. Interferon-β (IFN-β) therapy decreases the frequency of relapses, especially moderate and severe attacks. As judged by serial MRI studies, disease activity is lessened with IFN-β treatment.

o c. Glatiramer acetate also decreases the frequency of relapses, especially for patients with mild disease.

o d. Otherwise, treatment is directed at symptoms.1. (1) Modafinil, amantadine, and pemoline can improve fatigue.2. (2) Baclofen, tizanidine, and diazepam improve spasticity.3. (3) A variety of agents may help urologic dysfunction,

depending on the specific problem.o e. Patients with severe disease may respond to immunosuppressive

therapy.

Clinical trial result guide recommendations of therapy, which should be offered to pts 18 yrs or older, who are ambulant and have no contraindications to therapy, in the following clinical situations:

1. Clinically isolated syn. (one episode) and abnormal MRI typical of MS should be offered IFN-β (reduces relapses by about 1/3 over2 yrs) if they are within 1 yr of presentation.

2. Pts with relapsing-remitting MS and active disease (2 or more relapses in the last 2 yrs, or 1 disabling relapse, or MRI with new lesions in the last yr) should be offered IFN-β or Glatiramer acetate (reduce the relapse rate by 1/3 in relapsing-remitting MS).

Page 55: Neurology

In pts with relapsing secondary progressive MS, treatment (Glatiramer acetate, and IFN-β reduce relapses) should only be considered when relapses are the dominant cause of increasing disability. Treatment for primary progressive MS is not recommended. Decisions on the D/C of treatment are generally made clinically, on the basis of:

Increasing severity of relapses. Lack of relapse reduction on treatment compared to pre-

treatment. The development of non-relapsing secondary

progressive MS with loss of ability to walk.Neutralizing antibodies can develop to both interferon IFN β-1b (40% of pts) and IFN β-1a (20% of pts). These ABs are associated with a reduction in drug effect on relapse rate and also MRI lesions; in some pts the Abs disappear subsequently, but in others they persist. It is unclear whether neutralizing ABs influences the progression of disability in any way. These treatments modify the disease and may reduce the development of disability through preventing relapses, although any effect has, to date, been modest. They do not affect progression of disability that is unrelated to relapse. Other immunomodulatory agents for MS treatment:

Copolymer 1(Glatiramer acetate) and IV immunoglobulin therapy both significantly reduce the frequency of attacks of relapsing-remitting MS.

Oral low-dose Methotrexate therapy also slows down the progression of disability in secondary progressive MS(and possibly in primary progressive MS)

A number of other treatments, e.g. Mitoxantrone and Natalizumab are under study.

B. Central pontine myelinolysis

1. Etiology. Demyelination of the pons and deep cerebral white matter has been associated with the excessively rapid correction of severe hyponatremia. Central pontine myelinolysis has been seen in alcoholics, malnourished patients, and in association with diuretic use.

2. Diagnosis . Patients develop impaired arousal, quadriparesis, and pseudobulbar signs. MRI or CT scan confirms the diagnosis.

3. Therapy. Consists of supportive measures. Hyponatremia should be corrected slowly, and hypernatremia should be avoided.

XII. Myelopathy and Other Spinal Cord DisordersA. Definition

Page 56: Neurology

Myelopathy refers to a disorder of the spinal cord. Specific syndromes occur in relation to the various motor and sensory tracts affected by the different disease processes (Figure 11-4).B. Etiology

Selected causes of myelopathy are listed in Online Table 11-8.ONLINE TABLE 11-8 Selected Causes of Myelopathy

Vertebral column disorders   Trauma   Cervical stenosis   Disk protrusion   Odontoid subluxation   Rheumatoid arthritis   Down syndromeNeoplasia   Epidural spinal cord compression   Intradural extramedullary mass   Intramedullary massInfection   Spinal epidural abscess   Human immunodeficiency virus (HIV)   Human T-cell lymphotrophic virus type I (HTLV-I)Multiple sclerosisVascular disease   Infarction   Vascular malformationMetabolic diseases   Vitamin B12 deficiency (subacute combined degeneration)   Vitamin E deficiency   AdrenomyeloneuropathyRadiation effectsSyringomyeliaSpinocerebellar degeneration

C. Clinical signsDisease affecting the spinal cord can present with principally “long tract signs” or a combination of long tract signs and local radicular features.

1. Long tract motor signs result from the disruption of descending corticospinal fibers, which causes weakness, spasticity, and hyperreflexia.

2. Impaired sensation results from disordered function of ascending spinothalamic (pain and temperature) and dorsal column pathways (vibration and proprioception) (see Figure 11-4).

3. Bowel, bladder, and erectile function may be compromised. 4. Local radicular symptoms and signs include radiating pain, weakness, and

sensory loss referable to one or several myotomes and dermatomes. These findings help define the rostral–caudal extent of a lesion (Figure 11-5).

D. Selected conditions

1. Syringomyelia

Page 57: Neurology

o a. Definition . Syringomyelia is a condition of unknown cause resulting in cavitation of the spinal cord. Syringomyelia can occur in isolation or in association with the Arnold-Chiari malformation (i.e., descent of the cerebellar tonsils into the cervical spinal canal).

View Figure

FIGURE 11-4 Cross section of the spinal cord at C1. CST, corticospinal tract; DC, dorsal column; STT, spinothalamic tract.

o P.550o

FIGURE 11-5 Topographic relationship among nerve roots, spinal cord segments, and the bodies and Spinous processes of the vertebrae, which are indicated by Roman numerals.

Page 58: Neurology

View Figure

o b. Diagnosis (1) Signs of lower motor neuron dysfunction develop on a

segmental basis and are accompanied by upper motor neuron signs caudal to the cavity.

(2) Dermatomal loss of pain and temperature sensibility results from local disruption of the crossing spinothalamic fibers by the cavity (syrinx). Impaired ascending dorsal column sensibility occasionally occurs caudal to the cavity.

(3) MRI of the spine is the optimal

diagnostic test ( Online Figure 11-6).

View Figure

ONLINE FIGURE 11-6 Nonenhanced T1-weighted magnetic resonance imaging (MRI) scans demonstrating syringomyelia and an associated Arnold-Chiari malformation.

o c. Therapy. Surgery is occasionally indicated to decompress the fluid-filled spinal cord cavity and to perform a biopsy of the wall to evaluate the possibility of a cavitary neoplasm. Otherwise, treatment is supportive.

2. Transverse myelitis . Inflammation of the spinal cord can cause an acute myelopathy.

o a. Etiology. Although many cases of segmental transverse inflammation are idiopathic, MS, SLE, and various infectious agents may cause transverse myelitis.

o b. Diagnosis. Patients may experience localized back or radicular pain, followed by prickling or burning sensations and progressive weakness in the legs. Bowel and bladder disturbances are usually present.

o c. Therapy. Corticosteroid treatment is often advocated but is of unproven value.

3. The Brown-Séquard syndrome is caused by a lesion of one side (lateral hemisection) of the spinal cord at a discrete level. There is caudal ipsilateral upper motor neuron weakness below the lesion (severed descending corticospinal tract fibres), upper motor neuron signs, and loss of

Page 59: Neurology

proprioception (joint position sense) and vibratory sensation (severed ascending dorsal column fibres) as well as contralateral loss of pain and temperature sensation (severed crossed ascending spinothalamic tract fibres).

4. Anterior spinal artery occlusion o a. Etiology. Blockage of a radicular artery to the spinal cord can cause

an ischemic infarction. (1) In many cases, the artery of Adamkiewicz, a branch of the

aorta supplying the anterior two-thirds of the lumbar spinal cord, is compromised.

(2) Thrombosis can be caused by aortic dissection, local atherosclerosis, vasculitis, and hyperviscosity.

o b. Diagnosis. Patients present with flaccid, hyporeflexic paraplegia; impaired lower extremity pain and temperature sensation; and compromised bladder and bowel function. However, position and vibration senses are usually preserved.

o c. Therapy. Treatment of the disease responsible for the spinal cord infarction.

XIII. NeuropathyA. ClassificationNeuropathies may be classified by:

1. Course (acute, subacute, or chronic) 2. Type of symptoms and signs (sensory, motor, autonomic, or any

combination of the three) 3. Presence of pain (hyperesthesia or dysesthesia) 4. Distribution (generalized, focal, or multifocal) 5. NCV/EMG features (axonal or demyelinating)

B. EtiologySelected causes of neuropathy are listed in Tables 11-9A and 11-9B.C. TherapyControl of the underlying disease process is critical.

1. If impaired sensation renders the patient prone to injury, protective measures should be taken.

2. Weakness (e.g., wrist or foot drops) calls for appropriate splinting and physical therapy.

3. Autonomic insufficiency is difficult to manage; orthostatic hypotension can be treated with agents that expand blood volume (e.g., fludrocortisone) and increase vascular tone (e.g., midodrine).

4. Tricyclic antidepressants, carbamazepine, phenytoin, and gabapentin, can help patients with pain.

D. Selected syndromes

1. Compression neuropathies o a. Pathophysiology.

Page 60: Neurology

(1) Nerves can be damaged by repeated wear against firm surfaces, typically bone or fibrous tissue. Motor and sensory loss develops referent to the affected nerve.

(2) Common sites of compression leading to focal nerve dysfunction include the median nerve at the wrist (carpal tunnel), the ulnar nerve at the elbow, and the peroneal nerve at the fibular head.

o b. Etiology. The carpal tunnel syndrome can result from repetitive wrist movements, trauma, carpal tunnel stenosis, arthritides (rheumatoid arthritis and crystal-induced synovitis), diabetes mellitus, myxedema, pregnancy, birth control pills, acromegaly, and infiltrative processes such as amyloidosis.

o c. Diagnosis. A positive Tinel's sign (eliciting paresthesias by percussing the nerve at the site of compression) may be seen clinically. An NCV study reveals evidence of focal demyelination. If compression is severe, evidence of axonal injury may appear (e.g., muscle wasting, denervation on the EMG).

o d. Therapy. Treatment of underlying conditions is important. Splinting often alleviates the condition, especially if aggravating maneuvers can be eliminated. Local corticosteroid injections may be beneficial. Surgical decompression of the nerve is necessary at times.

P.552

TABLE 11-9A Selected Types and Causes of Neuropathy

Neuropathy

Causesacutesubacute or

chronicchronic

Sensory neuropathy

 Diabetes mellitusUremiaAlcohol abuseDeficienciesVitamins B1, B6, B12, niacinHIVHereditary neuropathiesDrugsVinca alkaloidsCisplatinPhenytoin2′,3′-Dideoxycytidine

ToxinsVitamin B6 intoxicationSjögren's syndrome*Paraneoplastic (anti-Hu antibody)*ParaproteinemiaCryoglobulinemiaAmyloidosisLeprosy

Motor neuropathy

Guillain-Barré syndromeDiabetes mellitus (proximal

CIDPLead intoxicationMultifocal motor neuropathy

 

Page 61: Neurology

ischemic neuropathy)Critical illness polyneuropathyPorphyria

Antibodies to GM1

Charcot-Marie-Tooth disease

Sensorimotor neuropathy

 Diabetes mellitusUremiaCritical illness polyneuropathyVasculitisHypothyroidismLyme diseaseParaproteinemia†CryoglobulinemiaParaneoplasticDrugsToxins

CIDP/DADS†Charcot-Marie-Tooth diseaseOther hereditary neuropathiesMetachromatic leukodystrophyRefsum diseaseAdrenomyeloneuropathyLipoprotein deficienciesSarcoidosis

Autonomic neuropathy

Guillain-Barré syndromePorphyria

Diabetes mellitusAmyloidosisFamilial dysautonomia

HIVVincristine

HIV, human immunodeficiency virus; CIDP, chronic inflammatory demyelinating polyneuropathy; DADS, distal acquired demyelinating symmetric neuropathy.*Also, sensory neuronopathy.†Including myelin-associated glycoprotein (MAG) antibody and monoclonal gammopathy.

2. Guillain-Barré syndrome o a. Definition and etiology. Guillain-Barré syndrome is a predominantly

demyelinating motor polyneuropathy that usually occurs in otherwise healthy individuals. The illness can follow a nonspecific viral syndrome or be associated with HIV infection, Campylobacter jejuni infection, hepatitis, infectious mononucleosis, Mycoplasma pneumoniae infection, vaccination, surgery, lymphoma, or SLE.

o b. Diagnosis (1) Clinical signs

(a) Classically, patients present with progressive weakness and areflexia. Progression of the disease should not extend beyond 4–6 weeks.

(b) Generalized paralysis can develop gradually or relatively acutely, impeding respiratory function.

(c) Relatively minor sensory signs and symptoms occur; however, patients may complain of painful extremities.

(d) The autonomic nervous system is often involved. Involvement of the autonomic nervous system can lead to early mortality as a result of cardiac arrhythmias and wide swings in blood pressure.

Page 62: Neurology

P.553

TABLE 11-9B Selected Types and Causes of Neuropathy Classified by Other Features

NeuropathyCausesDysesthetic neuropathyDiabetes mellitus

Alcohol abuseHIV2′,3′-DideoxycytidineVasculitis

Axonal neuropathyDiabetes mellitusUremiaDrugs and toxinsCritical illness polyneuropathyVasculitisParaproteinemiaCryoglobulinemiaVitamin B12 deficiencyHereditary neuropathies

Demyelinating neuropathyGuillain-Barré syndromeCIDPParaproteinemia*Hereditary neuropathies

Multifocal (mononeuritis multiplex) neuropathy

Diabetes mellitusVasculitisLyme diseaseLeprosySarcoidosisHereditary liability to pressure palsyMalignant infiltratesHIV, human immunodeficiency virus; CIDP, chronic inflammatory demyelinating polyneuropathy.*Including myelin-associated glycoprotein (MAG) antibody and monoclonal gammopathy.

(2) Diagnostic studies (a) Examination of the CSF shows an elevated protein

and less than 50 mononuclear cells/mm3 (albuminocytologic dissociation).

(b) The motor nerve conduction velocities are typically slowed.

(c) An abnormally small compound muscle action potential amplitude obtained with distal stimulation of a peripheral nerve (a measure of the integrity of the most

Page 63: Neurology

distal parts of the axonal portion of the nerve) is associated with a poor prognosis.

o c. Therapy (1) Plasmapheresis can shorten the length of time that patients

are dependent on a respirator and unable to ambulate. Criteria to initiate plasmapheresis include the inability to walk or rapid progression of the disease.

(2) Intravenous immunoglobulin treatment is also efficacious and is better tolerated than plasmapheresis.

3. Diabetic neuropathy (see also Chapter 9 IV A 7 d). Diabetes mellitus causes several neuropathic syndromes. The nerve injury may be secondary to chronic hypoxia (related to microvascular disease) that leads to axonal damage.

o a. Types. (1) A predominantly sensory, distal, symmetric, small fiber

polyneuropathy can be dysesthetic and involve pain and temperature modalities more than vibration and position senses.

(2) A predominantly sensory, distal, symmetric, large-fiber polyneuropathy may occur, affecting vibration and position modalities.

(3) A sensorimotor neuropathy can develop. (4) An autonomic neuropathy or a mononeuropathy or

mononeuritis multiplex can occur.

P.554

(5) Proximal diabetic neuropathy represents injury to large nerves that causes weakness and pain; it commonly involves the lumbosacral plexus.

o b. Therapy. As a group, patients with better blood glucose control have a less severe polyneuropathy. However, in individual patients, symptoms usually do not respond to tighter blood glucose control.

4. Chronic inflammatory demyelinating polyneuropathy (CIDP). This condition, which can be idiopathic or associated with a monoclonal gammopathy, causes sensorimotor neuropathy. Antimyelin-associated glycoprotein antibodies may be detected or there may be an IgM monoclonal gammopathy. Several variants of CIDP have been defined based on characteristic clinical presentations, associated monoclonal proteins or antibodies, and response to therapy.

o a. Diagnosis. NCV studies indicate slowing, and the CSF total protein is elevated.

o b. Therapy. CIDP is responsive to corticosteroid therapy, plasmapheresis, and intravenous immunoglobulin administration, as well as several immunosuppressant drugs.

XIV. Disorders of the Neuromuscular JunctionA. Myasthenia gravis

Page 64: Neurology

1. Etiology. Antibodies directed against the acetylcholine receptor on the muscle surface cause an increased rate of receptor destruction and lead to weakness.

2. Diagnosis o a. Clinical signs. Patients are often young women or older men.

Complaints of double vision, difficulty swallowing and speaking, and limb weakness and fatigue is common. On examination, ptosis, eye movement abnormalities, and proximal weakness may be seen. A thymoma is present in 10%–25% of patients.

o b. Diagnostic studies (1) Administration of intravenous edrophonium (a short-acting

cholinesterase inhibitor) usually produces a transient improvement in strength in patients suffering from myasthenia gravis. Patients with respiratory compromise or excessive oral secretions should not be given edrophonium because they may be unable to compensate for the increase in secretions that occurs after administration of this agent.

(2) Repetitive nerve stimulation studies can demonstrate a decremental response of the compound muscle action potential.

(3) Acetylcholine receptor antibodies can be detected in the blood of 80%–90% of patients.

(4) A thoracic CT or MRI scan shows a thymoma, if present. 3. Therapy

o a. The mainstay of therapy is administration of a cholinesterase inhibitor (e.g., pyridostigmine).

o b. Thymectomy can often lead to improvement; the presence of a thymoma is a definite indication for surgery.

o c. Corticosteroids, immunosuppressive agents, intravenous immunoglobulin, or plasmapheresis are effective in patients with refractory disease.

B. Eaton-Lambert myasthenic syndrome

1. Pathophysiology. This syndrome results when antibodies directed against the calcium channels on the presynaptic membrane of the neuromuscular junction interfere with the calcium-mediated release of acetylcholine vesicles in response to nerve stimulation. This syndrome is often associated with an underlying malignancy, especially small cell carcinoma of the lung, and autoimmune diseases.

2. Diagnosis o a. Clinical signs. Patients complain of weakness and fatigue, have

diminished muscle stretch reflexes, and may have impaired autonomic function, leading to dry mouth and poor visual accommodation. Unlike myasthenia, strength improves with activity.

o b. Diagnostic studies. Repetitive nerve stimulation studies show an incremental response of the compound muscle action potential.

3. Therapy. Diaminopyridine, plasmapheresis, intravenous immunoglobulin, or immunosuppressive therapy may be helpful.

P.555

Page 65: Neurology

XV. Disorders of MuscleA. Muscular dystrophies

1. Types of muscular dystrophies o a. Duchenne muscular dystrophy (DMD). is an X-linked recessive

disorder caused by a defect in the dystrophin gene, which codes for a muscle membrane protein, dystrophin, that is not detectable in patients with DMD.

(1) Although distal muscles are eventually affected as well, patients experience initial progressive proximal muscular weakness.

(2) Prednisone treatment slows progression of weakness. (3) Death usually occurs in the third decade of life, often as a

result of pneumonia.o b. Becker's muscular dystrophy (BMD) is an X-linked recessive

disorder of muscle caused by a defect in the dystrophin gene, which codes for a muscle protein, dystrophin, which is altered or present in reduced quantity in BMD. Patients with BMD have a more benign course than those with DMD, with a 50% survival rate beyond age 50.

o c. Myotonic muscular dystrophy is an autosomal dominant disorder localized to a gene, myotonin-protein kinase, located on chromosome 19. Patients with myotonic dystrophy have an increased number of CTG trinucleotide repeats in the kinase gene. The severity of the disease can increase in successive generations, and the CTG repeats increase proportionately (a phenomenon known as “anticipation”).

(1) Clinical signs include a characteristic muscle myotonia (i.e., persistent muscle activity in response to contraction or percussion), distal weakness, cataracts, frontal balding, impaired intellect, hypersomnia, testicular atrophy, cardiomyopathy, mitral valve prolapse, and cardiac conduction defects.

(2) Death occurs in the fifth or sixth decade and typically is attributable to respiratory compromise or cardiac arrhythmia.

o d. Facioscapulohumeral muscular dystrophy is an autosomal dominant disorder characterized by progressive weakness about the face, neck, upper torso, and proximal arms. The responsible gene is located on chromosome 4.

o e. Limb-girdle muscular dystrophy is actually a group of autosomal dominant and recessive disorders characterized by progressive loss of motor strength of the trunk and proximal limbs.

(1) The disorder represents a group of conditions with different genotypic origins and similar phenotype.

(2) Patients may have defects in their dystrophin-associated proteins (e.g. members of the dystrophin–glycoprotein complex) or other membrane-associated proteins.

2. Diagnosis. Patients typically have an elevated serum CK level. EMG demonstrates a “myopathic” pattern (i.e., brief, small amplitude muscle

Page 66: Neurology

potentials). A muscle biopsy is often informative. Genetic studies are increasingly pursued.

B. Acquired myopathy

1. Etiology. Muscle disease can be caused by inflammatory, toxic, or metabolic processes (Table 11-10).

2. Diagnosis o a. Clinical signs. A patient history and examination may provide clues

to the diagnosis. (1) Weakness is usually proximal and symmetric. Disease onset

can be acute, subacute, or chronic. (2) Swallowing and breathing can be compromised, and

myoglobinuria may result from rapid muscle destruction, leading to renal insufficiency.

o b. Diagnostic studies. The serum CK level may be elevated. The EMG reveals a “myopathic” pattern. A muscle biopsy is often informative.

3. Selected syndromes o a. Corticosteroid myopathy . Is usually caused by chronic

corticosteroid therapy and is associated with proximal muscle weakness and wasting. The serum CK level is normal, and the EMG is usually unremarkable.

o b. With polymyositis , the associated infiltration of lymphocytes destroys muscle fiber (see Chapter 10 IX D).

TABLE 11-10 Selected Causes of Acquired Myopathy Polymyositis   Idiopathic   Associated with other connective tissue diseases, infectious agents (including HIV), and drugsDermatomyositis   Associated with malignancyInclusion body myositisElectrolyte disorders   Hypokalemia   Hyperkalemia   Hypercalcemia   Hypomagnesemia   HypophosphatemiaEndocrine disorders   Hypothyroidism   Hyperthyroidism   Cushing's disease and iatrogenic corticosteroid administration   Addison's disease   AcromegalyDrugs (e.g., “statins” ε-aminocaproic acid, procainamide, zidovudine, phencyclidine, L-tryptophan)HIV, human immunodeficiency virus.

Page 67: Neurology

o c. Inclusion body myositis is an inflammatory myopathy characterized by a resistance to corticosteroid therapy and by distal and proximal weakness that may be asymmetric.

(1) Diagnosis. Muscle biopsy demonstrates inflammation and inclusion bodies (including “rimmed vacuoles”) that contain amyloid. Cytotoxic T (Tc) cells are active against a muscle antigen.

(2) Therapy. There is no accepted treatment.o d. Polymyalgia rheumatica (PMR) is not a myopathy but can

masquerade as one. (1) Clinical signs

(a) Patients often appear weak, but the major symptom is painful (tender, aching, and stiff) muscles.

(b) Activities such as ascending stairs may be difficult to perform. On formal strength testing, patients may appear slightly weak, presumably because the pain prevents maximal effort. It has been found that if the pain can be relieved, strength is preserved.

(c) Temporal (giant cell) arteritis is present in 15%–20% of patients.

(2) Diagnostic studies. The erythrocyte sedimentation rate usually is significantly elevated but may be normal. The serum CK level is normal, and the EMG is unremarkable. Muscle histology is normal; biopsy is usually not performed.

(3) Therapy. NSAID or corticosteroid therapy should bring about a rapid resolution of symptoms.

C. Nondystrophic myotonias (channelopathies)Nondystrophic myotonias (channelopathies) are characterized by prolonged muscle relaxation after voluntary contraction or mechanical stimulation.

1. Clinical signs. Online Table 11-11 summarizes the clinical features of the nondystrophic myotonias.

ONLINE TABLE 11-11 Clinical Features of the Nondystrophic Myotonias

Feature

Sodium Channel DiseasesChloride Channel Diseases

HyperkalemicParamyotoniaMyotonia CongenitaPeriodic Paralysis

CongenitaThompson's Becker's

Periodic paralysisYesYesNoYesPotassium-induced weakness

YesIn some families

NoNo

Cold-induced weaknessNoYesNoNoParadoxical myotoniaOccasionalYesNoNoProgressive weaknessVariableVariableNoRareSystemic involvementNoNoNoNoGenetic AD/17AD/17AD/7AR/7

Page 68: Neurology

transmission/chromosome locus

Adapted from Ptacek LJ, Johnson KJ, Griggs RC: Genetics and physiology of the myotonic muscle disorders. N Engl J Med 1993; 328(7):483.AD = autosomal dominant; AR = autosomal recessive.

2. Therapy. Treatment includes the use of quinine, procainamide, and phenytoin.

D. Metabolic myopathiesMetabolic myopathies include disorders of carbohydrate and lipid metabolism.E. MyoglobinuriaMyoglobinuria can be caused by crush injuries, vascular occlusions, infection, toxins, drugs, metabolic myopathies, hyperthermia, and severe inflammation.

1. Diagnosis is aided by documenting an elevated serum CK level and urinary myoglobin. The latter can be suspected on the finding of dark (brownish) urine and a positive dipstick for blood along with a paradoxical absence of RBCs in the urine.

2. Therapy should be directed at the underlying cause. Patients must be kept vigorously hydrated to prevent kidney damage.

F. Acute quadriplegic myopathyAcute quadriplegic myopathy can develop as a complication of critical illness and the systemic inflammatory response syndrome. There is an association with the use of corticosteroids and nondepolarizing neuromuscular blocking agents. Treatment is supportive.XVI. InfectionA. Meningitis, encephalitis, and neurologic complaints associated with HIV infectionA. Meningitis, encephalitis, and neurologic complaints associated with HIV infection are discussed in Chapter 8.B. Brain abscess

1. Etiology. An abscess can occur after neurosurgery or penetrating head trauma, in association with otitis media or poor oral hygiene, in patients with a bacteremia, and in individuals with a pulmonary arteriovenous malformation or cardiac right-to-left shunt.

2. Diagnosis o a. Clinical signs. Patients may present with headache, seizures, an

altered sensorium, and focal neurologic symptoms and signs.o b. Diagnostic studies. CT or MRI scanning can easily detect a brain

abscess, although differentiation from a neoplastic lesion can be difficult.

3. Therapy. Empiric treatment should be directed against aerobic and microaerophilic gram-positive streptococci and anaerobes, including Bacteroides fragilis.

o a. Antibiotic therapy. Often includes penicillin, chloramphenicol, metronidazole, and cefotaxime or trimethoprim–sulfamethoxazole. The clinical setting can help define the most likely pathogen.

Page 69: Neurology

o b. Therapy is monitored by serial brain imaging techniques.o c. Often, surgical excision of the abscess can be avoided. Stereotactic

biopsy and drainage of an abscess occasionally is required.

C. A spinal epidural abscessA spinal epidural abscess is usually caused by hematogenous seeding of an infective organism and is characterized by local pain and tenderness, fever, and neurologic signs and symptoms appropriate to the site of the infection.

1. Diagnosis can be aided by spinal MRI or CT scans and myelography. 2. Surgical drainage is indicated along with antibiotic therapy.

D. Neurosyphilis

1. Stages and clinical signs o a. Asymptomatic disease. CSF abnormalities may be the only sign of

infection.o b. Acute syphilitic meningitis usually develops within 2 years of

primary infection. Headache, meningismus, hydrocephalus, and cranial nerve palsies can occur.

o c. Cerebrovascular (meningovascular) syphilis manifests months to years after primary infection. Affected patients present with ischemic strokes (often associated with headache) and behavioral abnormalities.

o d. General paresis develops one to two decades after primary infection and is characterized by a progressive dementia.

o e. Tabes dorsalis also manifests 10–20 years after primary infection with lightning pains, paresthesias, bladder dysfunction, gait instability, Argyll Robertson pupils (i.e., impaired pupillary light reaction with preserved pupillary constriction to accommodation, perhaps as a result of a midbrain tegmental lesion), areflexia (especially at the ankles), and loss of position and vibration sensibility.

2. Diagnostic studies. Patients with HIV infection are at particular risk for neurosyphilis.

o a. Almost all patients with neurosyphilis have a reactive serum fluorescent treponemal antibody (FTA-ABS) test. Patients with a nonreactive CSF FTA-ABS test do not have neurosyphilis.

o b. The CSF VDRL test is often reactive in neurosyphilis, but a nonreactive CSF VDRL test is found in 25-75% of patients with neurologic disease years removed from the primary infection.

o c. Other CSF findings include a predominantly mononuclear pleocytosis and an elevated protein and IgG index.

3. Therapy. Penicillin is the treatment of choice for neurosyphilis. Patients with concurrent HIV infection develop neurosyphilis earlier and may be more resistant to therapy than immunocompetent patients.

XVII. Primary CNS TumorsAlthough these tumors are relatively uncommon, they seem to be increasing in incidence.A. Astrocytic neoplasms

Page 70: Neurology

1. Astrocytomas are neoplasms with slight hypercellularity and pleomorphism. Anaplastic astrocytomas are characterized as having moderate cellularity and pleomorphism and some vascular proliferation.

o a. Clinical signs. Patients with astrocytomas and anaplastic astrocytomas typically present with focal hemispheric neurologic dysfunction, convulsions, or headache.

o b. Therapy. Anaplastic astrocytomas can be treated with surgery and radiotherapy. Chemotherapy may be helpful.

o c. Prognosis. Outcome is inversely related to the patient's age and the presence of tumor necrosis. The survival rate after 2 years is 38%–50%. Other predictors of outcome include the patient's functional status and the amount of residual tumor after initial surgery.

2. Glioblastoma multiforme has moderate-to-marked hypercellularity, pleomorphism, and necrosis; vascular proliferation may be present.

o

a. Clinical signs. Clinical features are similar to those of less aggressive tumors. CT or MRI scans typically show an enhancing,

irregular mass ( Online Figure 11-7).

View Figure

ONLINE FIGURE 11-7 An enhanced T1-weighted magnetic resonance imaging (MRI) scan demonstrating a glioblastoma multiforme.

o b. Therapy. Treatment includes corticosteroid therapy (for edema reduction), surgical debulking, radiation therapy, and chemotherapy, both systemically and locally administered.

o c. Prognosis. The prognosis of glioblastoma multiforme is poor, with a survival rate of 10% after 24 months.

B. Oligodendrogliomas

1. Clinical signs. Patients with these infiltrating tumors commonly present with headache and convulsions; focal neurologic deficits can develop.

Page 71: Neurology

2. Therapy. Anaplastic oligodendrogliomas seem to be responsive to radiation therapy and chemotherapy. Surgery is the mainstay of treatment.

3. Prognosis. The overall median length of survival is 53 months.

C. MeningiomasMeningiomas are tumors that arise from the meninges and slowly enlarge; causing a mass effect that displaces normal structures. Angioblastic meningiomas are locally invasive.

1. Clinical signs. Headache, seizures, and focal neurologic signs can occur. 2. Therapy. Treatment is surgical resection; radiation therapy can be used for

invasive tumors. 3. Prognosis. If the entire tumor can be surgically resected, the majority of

patients do well. If the entire tumor cannot be removed, the patient may experience recurrence of symptoms.

D. SchwannomasSchwannomas of the eighth cranial nerve (acoustic neuroma) typically arise from the vestibular component of the nerve. They can enlarge and displace structures about the cerebellopontine angle.

1. Clinical signs. Patients develop dizziness, hearing loss, and tinnitus. A diminished corneal reflex may be a sign of trigeminal nerve compromise by an enlarging mass.

2. Diagnostic studies. Diagnosis is best made with an enhanced MRI. 3. Therapy. Treatment options include surgical resection or stereotactic

radiation therapy. 4. Prognosis. Small tumors can be surgically cured. If residual tumor remains,

recurrent symptoms can develop, usually years later.

E. Primary CNS lymphomaPrimary CNS lymphoma is increasing in incidence, especially in immunocompromised patients. Occasionally, primary CNS lymphoma presents as meningeal lymphomatosis.

1. Diagnostic studies. Imaging studies show one or more intensely enhancing lesions, typically in a periventricular distribution. The diagnosis can be established with stereotactic biopsy.

2. Therapy. Although initial treatment with corticosteroids can lead to a rapid decrease in the size of the mass, ultimate survival depends on radiation therapy and chemotherapy.

3. Prognosis. The development of multimodality treatment protocols has extended patient survival for years, especially in immunocompetent patients.

XVIII. Hereditary DisordersA. Wilson's disease (hepatolenticular degeneration)

This autosomal recessive disease is localized to an abnormality of the ATP7B gene on chromosome 13.the gene sequence is similar to sections of the gene ATP7A, which is defective in Menke's disease (another disease caused by defects in copper transport).the similar sequences code for copper-binding regions, part of a P-type

Page 72: Neurology

ATPase transmembrane protein. It may be associated with the accumulation of copper in the brain, liver, and other tissues (the eye). Abnormalities in the copper-binding protein ceruloplasmin are usually found.

1. Clinical signs. Presenting signs may include hepatic disease; Kayser-Fleischer rings in Descemet's membrane of the cornea; or neurologic symptoms: behavioral problems (including psychosis), movement disorders (e.g., incoordination, tremor, masked facies, dystonia, and athetosis); and hemolytic anemia. There are also “presymptomatic” patients who may present without these symptoms.

2. Diagnosis: this disorder should be considered in any young person presenting with an Extrapyramidal syn., psychiatric symptoms are particularly common in adults.

o a. Diagnosis is made by finding a low serum ceruloplasmin level (usually), excessive 24-hour urine copper excretion, Kayser-Fleischer rings on eye examination with a slit lamp, or increased levels of hepatic copper (by biopsy).

o b. MRI may reveal atrophy of the caudate and putamen with increased signal intensity on T2-weighted images.

3. Therapy. Treatment consists of copper chelation with trientine or D-penicillamine, if copper levels are high enough. Zinc may be used both as a maintenance therapy or for presymptomatic patients.

B. Neurofibromatosis (NF)

1. Neurofibromatosis type 1 (NF 1, von Recklinghausen's disease) is an autosomal dominant disorder. The responsible gene is located on chromosome 17, and the gene product is neurofibromin.

o a. Patients who meet two or more of the following criteria can be diagnosed as suffering from NF 1.

(1) Neurofibromas (two or more, or one plexiform neurofibroma)

(2) Café-au-lait macules (six or more measuring 1.5 cm in their greatest dimension)

(3) Freckling in the axillary or inguinal areas (4) Optic glioma (5) Two or more iris hamartomas (Lisch nodules) (6) Sphenoid dysplasia or thinning of the cortex of the long

bones (7) An immediate relative with NF 1

o b. Complications of NF 1 include astrocytic tumors, optic glioma, neurofibrosarcoma, compressive peripheral neuropathies, compressive myelopathy, pheochromocytoma, and scoliosis.

o c. Therapy is directed at the complications of the disease. 2. Neurofibromatosis type 2 (NF 2) is an autosomal dominant disorder

localized to chromosome 22 and characterized by bilateral acoustic neurofibromas.

Page 73: Neurology

C. von Hippel-Lindau diseaseThis autosomal dominant disorder is localized to chromosome 3. Cerebellar and retinal hemangioblastomas are characteristic.

1. Hemangioblastomas also may be found throughout the CNS. The kidneys, pancreas, and liver can harbor hemangiomas. Pheochromocytomas may develop.

2. Polycythemia is associated with cerebellar hemangioblastoma.

D. Osler-Weber-Rendu disease (hereditary hemorrhagic telangiectasia)This autosomal dominant disorder is associated with two genes (HHT1 and HHT2). It is characterized by telangiectatic skin lesions, CNS vascular malformations, and pulmonary arteriovenous fistulae. Brain abscesses may occur, as may ischemic stroke, secondary to a paradoxical embolus via a pulmonary arteriovenous fistula.

E. Tuberous sclerosisThis autosomal dominant disorder is associated with two abnormal genes, TSC1 and TSC2.

1. Criteria for diagnosis include multiple facial angiofibromas, ungual fibromas, retinal hamartoma, cortical tubers, subependymal glial nodules (often calcified), and multiple renal angiomyolipomas.

2. Some associated features are hypomelanotic macules, a shagreen patch, multicystic kidneys, seizures, and mental retardation. Benign giant cell astrocytomas can develop, often near the foramen of Monro.

F. Down syndrome (trisomy 21)This genetic disorder is characterized by mental retardation, epicanthal folds, and Brushfield spots on the iris, a transverse palmar crease, and cardiac malformations. By age 50, most patients develop Alzheimer's disease. Myelopathy may develop because of atlantoaxial dislocation.

G. Huntington's diseaseThe gene that is responsible for this autosomal dominant disorder, which is localized to chromosome 4, has an excess of trinucleotide repeats and encodes the protein huntingtin. Patients develop a progressive cognitive decline and choreoathetosis between 30 and 50 years of age. Depression frequently occurs. Atrophy of the caudate nucleus is characteristic.

H. Cerebellar atrophiesThese conditions are associated with multisystem degenerative processes including abnormal oculomotor function, bulbar wasting and fasciculations, upper motor neuron signs, Extrapyramidal dysfunction, cognitive decline, visual loss, and peripheral neuropathy.

1. Etiology. These diseases can be transmitted as autosomal dominant or recessive disorders, or they may be sporadic and presumably related to a new mutation.

2. Types. Numerous autosomal dominant spinocerebellar ataxias have been genetically identified. Most of these ataxias are characterized by an excess number of trinucleotide repeats and are identified as spinocerebellar atrophy type 1 (SCA-1), SCA-2, and so on.

Page 74: Neurology

o a. The phenotypic–genotypic correlations are often poor, making it difficult to accurately predict a patient's genotype from clinical evaluation alone.

o b. Molecular genetics will allow further genotypic classification of these disorders.

3. Associated conditions. Cerebellar atrophies have been associated phenotypically with deficiencies of glutamate dehydrogenase, pyruvate dehydrogenase complex, and hexosaminidase; vitamin E deficiency; mitochondrial disorders; and elevated very long–chain fatty acids.

I. Peroxisome disordersThe peroxisome is a cellular organelle that is involved with fatty acid oxidation.

1. Adrenoleukodystrophy is an X-linked disorder characterized by progressive intellectual decline, spasticity, and visual loss. Adrenomyeloneuropathy, also an X-linked disorder, is characterized by a progressive myelopathy and neuropathy.

o a. White matter changes are visible on CT and MRI scans in adrenoleukodystrophy.

o b. Both conditions result in excessive levels of very long–chain fatty acids in the blood, perhaps as a result of defective β-oxidation.

2. Refsum disease is an autosomal recessive disorder characterized by a sensorimotor neuropathy, pigmentary retinopathy, and ataxia, anosmia, hearing loss, skin lesions, and elevated CSF protein.

o a. Pathophysiology. There is an excess of phytanic acid in the blood because of a defect in alpha-oxidation of this fatty acid.

o b. Therapy. Dietary control of phytanic acid intake and plasmapheresis can help control the neurologic problems.

J. Mitochondrial disordersMitochondrial DNA codes for components of the mitochondrial respiratory chain and oxidative phosphorylation enzymatic complexes. Disorders of mitochondrial DNA are transmitted by nonmendelian, maternal inheritance. The mitochondrial encephalopathies and myopathies have ragged red fibers in muscle. The ragged red fibers represent abundant abnormal mitochondria that are demonstrated by using a modified Gomori trichrome stain. Patients may have sensorineural hearing loss, short stature, or diabetes mellitus, and they often have elevated blood lactic acid levels.

1. Kearns-Sayre syndrome is characterized by progressive external ophthalmoplegia, pigmentary retinopathy, and complete heart block, CSF protein levels greater than 100 mg/dL, ataxia, and myopathy. Deletions in the mitochondrial DNA are found.

2. Myoclonic epilepsy with ragged red fibers (MERRF) presents with myoclonus, epilepsy, and ataxia. There is a point mutation in the mitochondrial genome.

3. Mitochondrial encephalomyopathy with lactic acidosis and stroke-like events (MELAS) is characterized by intermittent vomiting and headaches, recurrent ischemic strokes, and seizures. Point mutations are present in the mitochondrial DNA.

Page 75: Neurology

4. Leber's hereditary optic neuropathy manifests as subacute bilateral central vision loss with retinal microangiopathy. It is caused by several point mutations in the mitochondrial genome.

XIX. Toxic and Metabolic DisordersA. Vitamin B12 deficiencyThis vitamin deficiency results in subacute combined degeneration or SCD.

1. Pathophysiology. Deprivation of vitamin B12 leads to demyelination and axonal degeneration, affecting the peripheral nerves, the spinal cord (where the posterior and lateral columns are demyelinated), and the cerebrum.

2. Diagnosis o a. Clinical signs. Neurologic manifestations include cognitive

impairment, diminished position and vibratory sensation, upper motor neuron signs with abnormal gait, and sensory peripheral neuropathy.

o b. Diagnosis. Patients typically have anemia, macrocytosis, and a low vitamin B12 level. However, neurologic disease can occur without anemia or macrocytosis. If the diagnosis remains suspect in a patient with a normal or marginally decreased serum vitamin B12 level, the finding of elevated serum methylmalonic acid and total homocysteine can confirm the presence of vitamin B12 deficiency.

3. Therapy. Treatment is administration of cobalamin.

B. Acute intermittent porphyria

1. Pathophysiology. This autosomal dominant disorder is caused by a defect in the activity of uroporphyrinogen I synthetase, which leads to increased activity of Δ-aminolevulinic acid synthetase and elevated levels of Δ-aminolevulinic acid.

2. Diagnosis. Acute intermittent porphyria causes delirium; seizures; and autonomic, sensory, and motor neuropathies; a Guillain-Barré–like illness can develop. During an acute attack, the Watson-Schwartz test indicates elevated levels of urinary porphobilinogen.

3. Therapy. Hematin administration can decrease clinical manifestations. Precipitants such as barbiturates, phenytoin, starvation, and infection should be avoided in susceptible individuals.

C. Complications of alcohol abuseAlcohol may affect the nervous system by itself (i.e., alcohol intoxication, addiction, or withdrawal) or in tandem with a nutritional deficiency.P.560

1. Complications from intoxication, addiction, and withdrawal o a. Acute alcohol intoxication. Causes delirium and incoordination.

(1) Severe intoxication can lead to metabolic coma with respiratory depression.

(2) Alcoholic “blackouts” are characterized by the inability to form new memories in spite of preservation of consciousness.

Page 76: Neurology

o b. Alcohol withdrawal causes early symptoms of tremulousness and hallucinosis. Delirium tremens is seen during late withdrawal from alcohol. Benzodiazepines may be used to treat alcohol withdrawal symptoms.

o c. Alcohol-related seizures (“rum fits”) are brief, generalized tonic–clonic convulsions that frequently occur in clusters.

(1) Classically, alcohol-related seizures were thought to occur primarily in the 48 hours after withdrawal from chronic alcohol intake. However, alcohol-related seizures also may be caused by chronic alcohol intoxication and represent a toxic effect of alcohol on the brain, not a withdrawal phenomenon.

(2) Typically, patients are alert shortly after the seizure. They do not have seizure discharges on an interictal EEG.

(3) Therapy should be directed at the underlying alcohol abuse. Anticonvulsant medications are not indicated for typical alcohol-related seizures.

o d. Alcohol abuse is associated with cerebral atrophy and cognitive impairments and may predispose patients to stroke.

o e. Alcoholic patients may develop a myopathy that is acute or chronic and predominantly affects the proximal muscles. Rhabdomyolysis can complicate the acute disorder.

2. Complications related to nutritional deficiencies o a. Wernicke's encephalopathy. Is seen in malnourished alcoholic

patients who have a thiamine deficiency. Other conditions (e.g., hyperemesis, renal dialysis, and malnutrition) are associated with thiamine deficiency and can lead to Wernicke's encephalopathy, even in patients who do not abuse alcohol.

(1) Clinical signs. Patients typically are delirious and have nystagmus, eye movement abnormalities, and ataxia. Disorders of consciousness and hypothermia can occur.

(2) Therapy. Thiamine administration can resolve the acute illness.

o b. Korsakoff psychosis is a chronic encephalopathy that is seen in alcoholic patients with a thiamine deficiency.

(1) Clinical signs. Features include retrograde and anterograde memory deficits, apathy, and impaired problem-solving abilities.

(2) Therapy. Patients may not improve with abstinence from alcohol or thiamine administration.

o c. Sensory neuropathy is caused by alcohol and often leads to dysesthesia. Although this axonal (although some authors suggest demyelinative) neuropathy is probably related to concurrent malnutrition, alcohol itself may be toxic to nerves.

o d. Cerebellar degeneration may occur in alcoholics. The anterior and superior vermis are particularly affected. Patients have difficulty with gait and stance; nystagmus and arm ataxia are not prominent.

D. Drug abuse

Page 77: Neurology

1. Use of cocaine or “crack” can result in seizures, ischemic stroke, subarachnoid and intraparenchymal hemorrhage, and rhabdomyolysis. Hemorrhagic strokes may be associated with an underlying vascular lesion; definition of the vascular anatomy is usually indicated to search for an aneurysm or arteriovenous malformation.

2. Use of heroin can cause a metabolic coma; miosis is a characteristic finding in heroin users. Heroin has been associated with rhabdomyolysis, chronic myopathy, transverse myelitis, and peripheral neuropathies.

3. Ingestion of phencyclidine (PCP) can cause a wide range of neurologic disturbances ranging from acute psychosis to coma. Nystagmus, miosis, ataxia, myoclonus, dystonic posturing, generalized rigidity, dyskinesias, and seizures can occur.

4. Ingestion of “ecstasy” (an amphetamine analog) can cause delirium, hyperthermia, and rhabdomyolysis. This drug selectively damages serotoninergic neurons.

P.561

XX. Sleep DisordersA. NarcolepsyNarcolepsy is characterized by hypersomnia with short latency periods for the onset of daytime sleep and the early development of rapid eye movement (REM) sleep. Patients awaken refreshed from sleep attacks.

1. Etiology. Narcolepsy is probably an autosomal dominant disorder with variable penetrance; it has been mapped to chromosome 6. The vast majority of patients have antigens for HLA-DR2 and HLA-DQB1.

2. Clinical signs. Features include cataplexy (i.e., transient episodes of diminished muscle tone), sleep paralysis, and vivid dreams at the beginning and end of sleep.

3. Therapy. Methylphenidate, pemoline, or modafinil can diminish the hypersomnia, and protriptyline or imipramine can decrease cataplexy.

B. Sleep apneaSleep apnea, or hypoventilation during sleep, can be attributable to obstructive causes (e.g., obesity, a small oropharynx) or to nonobstructive (CNS) causes. Sleep apnea is commonly “mixed” and represents both CNS and obstructive problems.

1. Clinical signs. Patients have daytime hypersomnia and can develop nocturnal hypoxia, pulmonary hypertension, and cardiac arrhythmias.

2. Therapy. Treatment options include weight loss, continuous positive airway pressure (CPAP), uvulopalatopharyngoplasty (UPPP), tracheostomy, and several new, minimally invasive surgical techniques.

C. Periodic movements in sleepPeriodic movements in sleep involve flexor contractions in the legs. Patients can also experience sleep myoclonus and dystonia and restless legs syndrome while awake.

Page 78: Neurology

1. Clinical signs. Patients are momentarily aroused, and therefore nocturnal sleep quality is poor, leading to daytime hypersomnia.

2. Therapy. Treatment options include carbidopa/levodopa combinations, dopamine agonists, gabapentin, opiates, and clonazepam.

XXI. TraumaA. Brain injury

1. A concussion is a momentary disruption of brain function after head injury that results in a brief loss of consciousness.

o a. Clinical signs. After awakening, patients complain of headache, impaired memory, poor concentration, blurred vision, tinnitus, dizziness, and nausea.

o b. Duration of symptoms. Symptoms can persist for days or weeks as the postconcussion syndrome. Posttraumatic migraine can develop but may be alleviated by using antimigraine agents.

2. Severe head trauma can cause brain contusion, epidural and subdural hematoma, penetrating brain injuries, SAH, and CSF leaks.

o a. Clinical signs. Progressive mental status changes leading to coma and focal neurologic signs can develop.

o b. Diagnostic studies. Emergent CT scanning can help delineate the extent of the problem.

o c. Therapy. Control of ICP and neurosurgical evaluation is indicated as appropriate.

3. A subdural hematoma can result from acute head trauma. Patients present with an altered sensorium and focal neurologic findings (e.g., hemiparesis). A subdural hematoma is also associated with minor trauma and can present with subtle subacute or chronic behavioral changes and mild focal deficits. At times, no history of trauma is elicited; this is especially true in elderly patients.

o

a. Diagnostic studies. A CT scan is very helpful in documenting

the presence of a subdural hematoma ( Online Figure 11-8).ONLINE FIGURE 11-8 A nonenhanced computed tomography (CT) scan showing a chronic subdural hematoma. Note the mass effect on the lateral ventricle and the absence of sulci.

Page 79: Neurology

View Figure

P.562

o b. Therapy. Neurosurgical intervention is usually indicated for an acute subdural hematoma; a chronic subdural hematoma may not require surgery.

B. Spinal cord injurySpinal cord injury can result in an acute paraparesis or quadriparesis. A central cord syndrome, which can follow a hyperextension injury to the cervical spinal cord, is characterized by lower motor neuron dysfunction affecting the cervical myotomes, mild sensory loss in the arms, and a myelopathy.

1. Diagnostic studies. Because the vertebral column may not be stable, patients should be immobilized while a radiographic assessment is made.

2. Therapy. High-dose methylprednisolone therapy given within 8 hours of injury can improve outcome.

P.563

Study Questions/Answers and Explanations1. A 70-year-old man, a retired professor, complains of weakness and fatigue. Any physical activity is an effort, and he cannot find a comfortable position at rest. He has lost 5 pounds over the past month. Physical examination is normal. Which of the following diagnoses is most likely?A. DepressionB. HypokalemiaC. Temporal (giant cell) arteritisD. Polymyalgia rheumatica (PMR)E. PolymyositisView Answer1. The answer is D [IV B 5 a; XV B 3 d (1)–(2)]. Polymyalgia rheumatica (PMR) is characterized by muscle discomfort, and patients often present with vague complaints, as in this case. By definition, the results of the neuromuscular examination are normal.Depression can cause fatigue and diminished activity, but discomfort is less frequently a complaint. A normal erythrocyte sedimentation rate, which is elevated in the presence of PMR, would support a diagnosis of depression. Hypokalemia can cause muscle weakness, but discomfort is not a typical feature. Temporal (giant cell) arteritis can occur with PMR, but this patient has no complaint of headache or symptoms referable to vision. Polymyositis can cause muscle discomfort, but there should be accompanying weakness. The serum creatine kinase (CK) level is often elevated.2. A 29-year-old woman suddenly develops a left hemiparesis. The patient experienced a deep venous thrombosis in her right leg 3 years ago. Which of the following conditions is the most likely cause of this patient's deficit?

Page 80: Neurology

A. Nonvalvular atrial fibrillationB. Lupus anticoagulantC. Mitral valve prolapseD. Multiple sclerosis (MS)E. AstrocytomaView Answer2. The answer is B [VIII B 6 b]. The lupus anticoagulant (an antiphospholipid antibody) is associated with peripheral venous thrombosis and ischemic (arterial) stroke. In patients with a history of deep venous thrombosis, the possibility of a paradoxical embolus causing a stroke (via a right-to-left cardiac shunt) should also be considered.Nonvalvular atrial fibrillation is a common cause of stroke in the elderly, but there is no reason to suspect a rhythm disturbance in this patient. Mitral valve prolapse has been associated with cardiogenic emboli and stroke. However, a search for other causes of stroke should always be made, because mitral valve prolapse is a relatively common entity, and the association with stroke is weak. Multiple sclerosis (MS) and an astrocytoma can cause neurologic deficits in young patients; however, these conditions are not associated with deep venous thrombosis unless the patient is immobilized. These diagnoses should be considered in the evaluation of patients with hemiparesis, but the patient's history can often serve as a clue to guide diagnostic thinking.3. An assembly-line worker complains of awakening at night with right-hand discomfort that resolves after several minutes. After 3 weeks of continuing symptoms, he seeks medical advice. Examination discloses mild weakness of thumb abduction and diminished pain sensibility on the palmar aspect of the thumb and index finger. Which of the following diagnoses is most likely?A. Carpal tunnel syndromeB. Cervical radiculopathyC. Reflex sympathetic dystrophyD. TendinitisE. Left middle cerebral artery ischemic attacksView Answer3. The answer is A [XIII D 1 a–b]. People such as assembly-line workers and typists are particularly prone to carpal tunnel syndrome because their daily activities require repetitive wrist movements, which may, in time, compress the median nerve, causing neuropathy. Awakening at night with hand discomfort is a common complaint, and the findings on examination support the diagnosis.A cervical radiculopathy can cause some of these findings. However, the history is more suggestive of carpal tunnel syndrome. Patients with a cervical radiculopathy often have a concurrent carpal tunnel syndrome and vice versa. The history and examination are not supportive of a diagnosis of tendinitis or reflex sympathetic dystrophy. Transient ischemic attacks (TIAs) rarely cause discomfort and are not associated with neurologic deficits after 24 hours have elapsed.4. A 73-year-old woman presents with a 6-month history of deteriorating gait and low back discomfort, which is exacerbated by walking. Examination is unremarkable except for hypoactive muscle stretch reflexes in the legs. Radiographs of the lumbosacral area show the expected degenerative changes associated with a woman of her age. Which of the following diagnoses is most likely?A. Acute lumbar disk herniationB. Lumbar stenosis

Page 81: Neurology

C. MyopathyD. Normal pressure hydrocephalus (NPH)E. Cervical stenosisView Answer4. The answer is B [VII A 2 b]. Lumbar stenosis is caused by degenerative changes in the lumbosacral spine, often in association with a congenitally small lumbosacral intraspinal space. The history is often that of vague low back discomfort associated with subtle findings on examination referable to impingement on motor and sensory roots. Diagnosis can be made by the characteristic finding of an “hourglass” appearance on magnetic resonance imaging (MRI) scans.An acute disk herniation is characterized by low back discomfort and pain extending in a radicular fashion down one or both legs. Examination is often consistent with impingement on a single sensory or motor root. A myopathy can cause an impaired gait; low back discomfort because of weakness; and hypoactive muscle stretch reflexes, typically at the knees. However, this condition is much less common than lumbar stenosis and therefore is not the most likely diagnosis. Normal pressure hydrocephalus (NPH) causes an apractic gait (i.e., difficulty in walking in spite of an intact motor, sensory, and cerebellar examination), cognitive impairment, and urinary incontinence. The clinical picture is not consistent with this diagnosis. Cervical stenosis can cause a myelopathy and resultant gait problem. Because the patient does not exhibit signs of a myelopathy, this diagnosis is unlikely.5. A 45-year-old right-handed man complains that he has had difficulty holding and using a writing instrument for the past year. He notes the development of right-hand and forearm spasms only when writing. Physical examination is unremarkable. Which of the following diagnoses is most likely?A. Parkinson's diseaseB. Focal dystoniaC. Carpal tunnel syndromeD. Cervical radiculopathyE. Benign essential tremorView Answer5. The answer is B [X B 1; X B 4 XIII D 1 a–b]. Writer's cramp is a focal dystonia of unknown cause. Patients develop symptoms of cramps or spasms with altered hand and arm posture when attempting the specific task of writing. Examination of the patient is otherwise normal.Micrographia is a symptom of Parkinson's disease but is usually accompanied by signs of rigidity and bradykinesia, and often tremor. Carpal tunnel syndrome is caused by pressure on the median nerve as it enters the hand via the carpal tunnel. Median nerve dysfunction leads to hand weakness and loss of sensibility, which can affect writing. A cervical radiculopathy can lead to hand numbness and weakness and hyporeflexia. The exact distribution of findings depends on the nerve roots involved. A benign essential tremor is characterized by a distal upper extremity tremor during a task. There is no accompanying rigidity or bradykinesia. Handwriting in particular may suffer. Carpal tunnel syndrome, cervical radiculopathy, and benign essential tremor are all unlikely because the patient's examination shows no signs or symptoms other than difficulty with writing.6. A 24-year-old construction worker with a 2-year history of low back pain complains of an acute onset of bilateral leg weakness and incontinence. Which of the following treatments would be the best management tactic?A. Administration of nonsteroidal anti-inflammatory drugs (NSAIDs)

Page 82: Neurology

B. Emergency surgeryP.564

C. Strict bed restD. Lumbar tractionE. Back exercisesView Answer6. The answer is B [VII A 2 a (2)–(3)]. The patient probably has an acute disk herniation causing a cauda equina syndrome, an indication for emergency surgery. Surgery should be considered as an elective intervention for those patients with sciatica, non-disabling neurologic deficits, and disk herniation (as demonstrated by appropriate imaging techniques) who fail to respond to 6 weeks of conservative management.Nonsteroidal anti-inflammatory drugs (NSAIDs) can be helpful in the treatment of acute, and possibly chronic, low back pain; however, they are not indicated as primary therapy for patients with acute, severe neurologic disability. Avoidance of strenuous activity is very helpful for the treatment of acute back pain, but this therapy is not appropriate for this patient. Lumbar traction is probably not effective for the treatment of low back pain. Patients typically cannot exercise during the first few days of acute back pain. However, as the acute pain subsides, an exercise program may help prevent future problems.7. A 32-year-old man presents with repetitive generalized motor convulsions that continue for 35 minutes until 2 mg of lorazepam are administered intravenously. The next course of action should be to administer which of the following?A. Phenytoin intravenouslyB. Carbamazepine orallyC. Pentobarbital intravenouslyD. Ethosuximide orallyE. Diazepam rectallyView Answer7. The answer is A [IX D 2 b (1)–(2), 4 b, c]. Administration of intravenous lorazepam should be followed by the administration of phenytoin (or fosphenytoin) to control status epilepticus because the duration of action of lorazepam is limited. Therefore, unless there is a contraindication, the patient should receive a loading dose of phenytoin (or fosphenytoin) intravenously.Carbamazepine is an effective anticonvulsant, but it cannot be given intravenously or intramuscularly. Therefore, a therapeutic level cannot be rapidly achieved given a drug half-life of 8–12 hours. Intravenous pentobarbital can be used to control repetitive seizures. However, because the patient is not currently convulsing, induction of barbiturate coma is not indicated. Ethosuximide is indicated for the treatment of absence seizures. Therefore, this is not an appropriate therapy for generalized motor convulsions. Rectal diazepam is used to abort seizures temporarily, especially in children.8. A 78-year-old woman complains of experiencing headaches and progressive confusion for the past month. She has a left hemianopia and cannot dress herself. A computed tomography (CT) scan demonstrates a large, irregularly enhancing mass in the right parietal lobe. There is no obvious systemic disease. Which of the following diagnoses is most likely?A. Brain abscess

Page 83: Neurology

B. Glioblastoma multiformeC. MeningiomaD. MetastasisE. Central nervous system (CNS) lymphomaView Answer8. The answer is B [XVII A 2]. A large, irregularly enhancing central nervous system (CNS) mass in an elderly patient without systemic cancer is highly suggestive of a glioblastoma multiforme. However, a biopsy is necessary before a definitive diagnosis can be made.The patient has no predisposing condition for a brain abscess such as poor dentition or intravenous drug abuse. A meningioma can occur in the parietal area and distort the brain; however, a “convexity” meningioma is typically a homogeneously enhancing lesion. In the absence of systemic cancer, a brain metastasis is an unlikely cause of the patient's problem. However, without a biopsy, the diagnosis of metastatic disease cannot be excluded. CNS lymphoma typically manifests as a homogeneously enhancing mass lesion. Patients with human immunodeficiency virus (HIV) and other immunocompromised patients are prone to CNS lymphoma; in other populations, CNS lymphoma is rare.9. A 63-year-old woman develops intermittent dizziness. Examination shows a diminished right corneal reflex and mild hearing loss in the right ear. Which of the following diagnoses is most likely?A. Cerebellopontine angle tumorB. Benign paroxysmal positional vertigo (BPPV)C. Lateral medullary syndromeD. Meniere's diseaseE. Pontine infarctionView Answer9. The answer is A [XVII D 1–3]. A cerebellopontine angle tumor such as a schwannoma can cause intermittent dizziness. The tumor can arise from the eighth cranial nerve, thereby also affecting hearing, and can press on the trigeminal nerve, causing impairment of the corneal reflex.Benign paroxysmal positional vertigo (BPPV) causes intermittent brief dizziness that is dependent on postural changes. Nystagmus is characteristic, but there are no other neurologic deficits. A lateral medullary syndrome usually causes constant dizziness that is exacerbated with movement. Although the corneal reflex can be depressed, hearing is normal. Meniere's disease causes intermittent dizziness and hearing loss, but the corneal reflex is not diminished. A pontine infarction does not cause intermittent symptoms; hearing loss is not expected as a result.10. A previously vigorous 80-year-old woman collapses when getting out of bed. Examination of her legs indicates bilateral weakness, loss of pain and temperature sensation, and areflexia. Her bladder is distended. The remainder of the examination is unremarkable. Which of the following diagnoses is most likely?A. Guillain-Barré syndromeB. Anterior cerebral artery occlusionC. Cauda equina syndromeD. Anterior spinal artery occlusionE. Thoracic spinal cord compressionView Answer10. The answer is D [XII D 4 b]. Thrombosis of the caudal anterior spinal artery leads to a flaccid paraplegia, loss of pain and temperature sensation, and bowel and bladder

Page 84: Neurology

dysfunction. The blood supply to the caudal spinal cord arises from the aorta via the lumbar Artery of Adamkiewicz. The perfusion territory of the anterior spinal artery involves the anterior horn cells and the pain and temperature pathways.In the Guillain-Barré syndrome, which rarely develops suddenly, loss of temperature and pain sensation can occur but rarely without a concurrent loss of position and vibration sense. Sensory loss is usually less severe than motor loss. Generalized areflexia is common. An anterior cerebral artery thrombosis, especially when bilateral, causes leg weakness, but sensory loss, when present, involves all modalities. Upper motor neuron signs are present in the leg. A cauda equina syndrome [VII A 2 (3) a] can cause a flaccid paraplegia, but all sensory modalities may be compromised because sensory nerve roots, which carry sensory fibers of all types, are involved in the disease process. Thoracic spinal cord compression is often associated with back pain and produces upper motor neuron dysfunction in the legs. In the acute condition, the bladder may be distended, but with time, its capacity is typically reduced.11. A 70-year-old man reports that over the past 2 months he has had progressive difficulty walking. Examination indicates distal (greater than proximal) weakness in the arms and legs and the absence of muscle stretch reflexes. Motor nerve conduction velocities (NCVs) are slowed. Which of the following diagnoses is most likely?A. Guillain-Barré syndromeB. Lead poisoningC. Chronic inflammatory demyelinating polyneuropathy (CIDP)D. Amyotrophic lateral sclerosis (ALS)E. PolymyositisView Answer11. The answer is C [XIII D D 4; Tables 11-9A and 11-9B]. Chronic inflammatory demyelinating polyneuropathy (CIDP) is characterized by progressive weakness that occurs over an extended time (e.g., >4 weeks), absent muscle stretch reflexes, and slowed motor nerve conduction velocities (NCVs).Guillain-Barré syndrome should not cause progressive worsening of strength beyond 4 weeks. Typically, only some motor nerves demonstrate slowing, usually in a segmental, rather than a generalized, fashion. Lead poisoning can cause a motor neuropathy in children but only rarely in adults. Amyotrophic lateral sclerosis (ALS) causes progressive weakness, but motor NCVs are unremarkable and muscle stretch reflexes are often hyperactive. Polymyositis causes progressive proximal (greater than distal) weakness and is not associated with impaired motor NCVs. Muscle stretch reflexes are often preserved in proportion to the muscle strength.12. A 56-year-old man presents with a 2-year history of impotence and not feeling “right.” Examination reveals masked facies, bradykinesia, rigidity, mild ataxia, and postural hypotension. The diagnosis is which of the following?A. Parkinson's diseaseB. Subacute combined degenerationP.565

C. Multiple systems atrophyD. Spinocerebellar degenerationE. Vitamin E deficiencyView Answer

Page 85: Neurology

12. The answer is C [VI A 1 b; X A 2 a (2); XIX A 2 a; online Table 11-7]. The signs and symptoms are most consistent with multiple systems atrophy (Shy-Dragger syndrome).The early appearance of autonomic dysfunction (impotence and postural hypotension) and ataxia do not support the diagnosis of Parkinson's disease. Combined systems degeneration is due to vitamin B12 deficiency. Parkinsonian features are not a feature of the disease. Spinocerebellar degeneration is characterized by cerebellar dysfunction and an accompanying upper motor neuron syndrome. Extrapyramidal and autonomic dysfunctions are not prominent features. Vitamin E deficiency can masquerade as spinocerebellar degeneration.13. A patient with a subarachnoid hemorrhage (SAH) caused by a right anterior communicating artery aneurysm undergoes successful surgery 2 days after the hemorrhage. Three days later, right arm weakness develops. Which of the following diagnoses is most likely?A. HydrocephalusB. MeningitisC. Repeat hemorrhageD. VasospasmE. HyponatremiaView Answer13. The answer is D [VIII C 1 c (2)]. Vasospasm can develop several days after an aneurysmal subarachnoid hemorrhage (SAH). Patients present with progressive weakness and alterations in consciousness. Early in the course, a computed tomography (CT) scan may not reveal an ischemic infarction.Hydrocephalus can occur immediately after an SAH or weeks to months later. Symptoms are typically nonfocal and, if the hydrocephalus develops acutely, it is often accompanied by a depressed level of consciousness. Bacterial meningitis can develop after a craniotomy. Typically, there is fever and impaired arousal. Focal signs can develop but are rarely the presenting feature. Although a repeat hemorrhage can occur after clipping of an aneurysm if the aneurysm is not completely isolated from the circulation, it is unusual for this to happen and present with a focal deficit, as opposed to depressed consciousness. Hyponatremia, which can develop after SAH, can cause an altered sensorium and seizures but not unilateral weakness.14. A 17-year-old high school varsity diver develops a headache, dizziness, left-sided arm and leg clumsiness, and loss of pain and temperature sensation in the left facial and right body areas after a practice session. Which of the following diagnoses is most likely?A. Benign paroxysmal positional vertigo (BPPV)B. Multiple sclerosis (MS)C. Vertebral artery dissectionD. AstrocytomaE. LabyrinthitisF. Migraine with auraView Answer14. The answer is C [VI B 2 f]. The patient has manifestations of a lateral medullary syndrome. In a young patient, who has subjected himself to strenuous neck movements, the most likely etiology is vertebral artery dissection.Benign paroxysmal positional vertigo (BPPV) causes sudden episodes of dizziness, typically with changes in position. However, there are no associated neurologic symptoms or signs other than nystagmus. Multiple sclerosis (MS) can cause dizziness

Page 86: Neurology

and clumsiness. However, the constellation of findings in this patient, which are referable to a single site within the brain and are of sudden onset, make the diagnosis of MS less likely. An astrocytoma can cause dizziness and unsteadiness. Often there is a headache. The onset is typically insidious, and the symptoms progressive. Labyrinthitis causes severe vertigo. Patients find it difficult to move about and prefer to remain still. Gait instability can occur because of the profound dizziness in the absence of cerebellar deficits. There are no neurologic signs other than nystagmus. Migraine with aura is associated with headache and focal neurologic symptoms and signs. Unless patients have a history similar to that described, they should be evaluated for alternative diagnoses such as arterial dissection. Patients with migraine associated with focal neurologic deficits should be evaluated for alternate diagnoses before the problem is attributed to migraine with aura.15. A 26-year-old woman presents with recurrent throbbing headaches accompanied by nausea, emesis, photophobia, and phonophobia. She has been treating herself with acetaminophen and ice packs to her forehead. A reasonable therapeutic intervention might be to prescribe:A. BaclofenB. AmitriptyleneC. IndomethacinD. CorticosteroidsE. SumatriptanView Answer15. The answer is B [IV B 1 d (1)–(2) (a)]. Migraine headaches respond well to treatment with tricyclic antidepressants such as amitriptylene. This patient has been unresponsive to over-the-counter acetaminophen. Assuming she is having frequent headaches that are interfering with her lifestyle, it is reasonable to begin treatment with a prophylactic migraine medication.Baclofen is not effective for migraine. Indomethacin is not commonly prescribed for migraine; though it may benefit an acute headache episode, it is not considered a prophylactic medication. Corticosteroids can be helpful in the treatment of a severe, acute migraine headache. However, corticosteroids are not commonly prescribed for migraine prophylaxis. Sumatriptan is an effective treatment for an acute migraine headache but it is not indicated as a prophylactic migraine therapy.16. A 76-year-old woman has been experiencing left temporal headaches for the prior 3 weeks. Over the prior 2 days she has had several brief episodes of cloudy vision in her left eye. On questioning, she notes a 5-pound weight loss over the past 2 weeks, which she attributes to discomfort when chewing. Therapy should be initiated with:A. BaclofenB. AmitriptyleneC. IndomethacinD. CorticosteroidsE. SumatriptanView Answer16. The answer is D [IV B 5 c]. This patient likely has giant cell (temporal) arteritis. The treatment of choice for giant cell arteritis is corticosteroids. An erythrocyte sedimentation rate and C-reactive protein assay should be obtained before beginning corticosteroid therapy, but waiting for a temporal artery biopsy should not delay initiation of therapy.Neither Baclofen, amitriptylene, indomethacin, nor sumatriptan are indicated for giant cell arteritis.

Page 87: Neurology

17. A 57-year-old man has a 2-month history of severe, daily headaches that involve the right frontotemporoparietal area. The headaches typically last 60–90 minutes and occur once or twice daily. Over-the-counter medications have not provided relief. Chiropractic manipulation did not help. The patient is desperate for relief because he cannot work during the headache episodes. A reasonable treatment strategy is to initiate:A. BaclofenB. AmitriptyleneC. IndomethacinD. CorticosteroidsE. SumatriptanView Answer17. The answer is C [IV B 8]. The patient's history is suggestive of paroxysmal hemicrania, an indomethacin-responsive disorder. In fact, a positive response to treatment with indomethacin can confirm a diagnosis of paroxysmal hemicrania.Baclofen, amitriptylene, corticosteroids, and sumatriptan are not considered beneficial for paroxysmal hemicrania.18. A 53-year-old woman with the sudden onset of the worst headache of her life presents to the emergency department. She has a history of migraine but states that the current headache is not like her P.566

Usual headaches. Results of her physical examination are unremarkable. The initial preferred diagnostic test is a (an):A. AngiogramB. CT scanC. TCDD. MRIE. MR angiogramView Answer18. The answer is B [VII C 1 b (2); IV A 3 b]. The history is highly suggestive of subarachnoid hemorrhage (SAH). A brain computed tomography (CT) scan is the best test to screen for intracranial hemorrhage and should be emergently obtained to document subarachnoid blood. If the scan is normal and the history is suggestive of SAH, the woman should undergo a lumbar puncture (LP) to screen for blood before the physician concludes that she does not have a SAH.Conventional angiography is necessary to evaluate patients with SAH to define the presence of an aneurysm and details of its anatomic configuration. Associated abnormalities such as vasospasm can also be assessed. However, an angiogram should not be the initial diagnostic test in a patient with a suspected SAH.Transcranial Doppler (TCD) studies can detect cerebral artery vasospasm but not the presence of an aneurysm. Vasospasm typically develops a few days after SAH; therefore, it would not be expected to be apparent at the time of patient presentation.Brain MRI is not as sensitive as brain CT for the detection of acute intracranial hemorrhage. Therefore, brain MRI is not considered the procedure of choice for the acute detection of SAH.Although magnetic resonance (or computed tomography) angiography (MRA or CTA, respectively) can document the presence of an aneurysm, they are not as sensitive as conventional angiography. Neither MRA nor CTA is adequate to

Page 88: Neurology

determine the characteristics of the aneurysm fully and to direct therapy. Therefore, MRA and CTA are not the first tests to be obtained in patients with a suspected SAH.19. A 78-year-old man with a history of type 1 (insulin-dependent) diabetes mellitus, hypertension, hypercholesterolemia, and remote smoking presents with the acute onset of aphasia. On examination, he has trouble expressing himself and has mild right facial and arm weakness. There is a left carotid bruit. The most definitive strategy for secondary stroke prevention will likely be:A. Acute heparin administration followed by warfarinB. Extracranial intracranial bypass surgeryC. Warfarin and aspirin combined therapyD. Carotid endarterectomyE. Aspirin monotherapyView Answer19. The answer is D [VIII B 4 c (1) (a), (3) (a)]. The patient has a nondisabling stroke. His risk factor profile and the presence of an ipsilateral bruit suggest that the most likely cause of his stroke is left internal carotid artery origin stenosis. The definitive intervention to prevent subsequent stroke is carotid endarterectomy.Acute heparin administration and subsequent warfarin therapy have not been demonstrated to be effective for secondary stroke prevention in this setting. Likewise, there is no role for extracranial-intracranial bypass surgery to treat extracranial internal carotid artery origin stenosis. A combination of warfarin and aspirin has not been shown to be more effective than carotid endarterectomy for secondary stroke prevention for a patient such as this. Although aspirin is appropriate therapy for secondary stroke prevention for stroke due to arterial disease, treatment with aspirin should not preclude carotid endarterectomy in the appropriate setting. Aspirin should be administered after carotid endarterectomy to decrease the risk of postoperative stroke.20. A 32-year-old woman who is 1 week postpartum presents with progressive headache and confusion. On examination she is afebrile with a blood pressure of 110/65 mm Hg. Papilledema is noted. The most likely diagnosis is:A. Pseudotumor cerebriB. Pituitary apoplexyC. Bacterial meningitisD. Sagittal sinus thrombosisE. EclampsiaView Answer20. The answer is D [IV B 6]. The patient's presentation suggests sagittal sinus thrombosis. Although this condition is associated with the hypercoagulable state of pregnancy, further testing is indicated to evaluate the possibility of an underlying chronic hypercoagulable condition.Pseudotumor cerebri (idiopathic intracranial hypertension) can present in association with pregnancy, and although it is characterized by headache and papilledema, confusion is not a part of pseudotumor cerebri.Pituitary apoplexy can occur postpartum. It is characterized by headache and visual dysfunction. Papilledema and confusion are not common sequelae of pituitary apoplexy.Although bacterial meningitis causes headache, confusion, and papilledema, patients are typically febrile.

Page 89: Neurology

Eclampsia can cause headache and confusion as well as papilledema. However, the patient's low blood pressure is evidence against this diagnosis. Also, there is no mention of proteinuria or seizures.21. A 67-year-old woman with a history of hypertension, diabetes mellitus, and smoking presents to the emergency department at 8:30 AM with a mild expressive aphasia, right facial weakness, and mild right arm weakness. She had awakened at 7:00 AM and was speaking to her husband when her speech suddenly became difficult and weakness was noted. Her husband called 911, and she was transported to the hospital, where her blood pressure was 165/85 mm Hg. The preferred treatment is:A. AspirinB. HeparinC. WarfainD. rt-PAE. ClopidogrelView Answer21. The answer is D [VIII B 1 c (1)]. The patient has had the acute onset of a neurologic deficit consistent with a stroke. She is presenting for medical care within 3 hours of symptom onset. No contraindications to intravenous thrombolytic therapy with rt-PA are mentioned. The patient should have an emergent brain CT scan and if no alternate diagnoses are suggested (tumor, subdural hematoma, etc.), intravenous rt-PA should be administered.Aspirin is standard of care if the patient had presented more than 3 hours after symptom onset. There is little evidence that heparin or warfarin therapy is beneficial in this setting, and there is an increased incidence of bleeding with these anticoagulants. Clopidogrel is a therapeutic alternative for secondary stroke prevention, but its use should not take precedence over the administration of rt-PA.If rt-PA is administered, aspirin administration should be delayed 24 hours. Also, if low-dose heparin is administered for deep venous thrombosis prophylaxis, its use should also be delayed for 24 hours before rt-PA use.22. A 62-year-old woman has a 2-month history of mild confusion. She occasionally has difficulty following a conversation. On the morning of presentation to the hospital, she developed 30 seconds of right face and arm twitching, followed by increased confusion. Examination shows difficulties in speech comprehension and a subtle right homonymous hemianopia. A brain CT scan shows a 2 to 3-cm ill-defined region of low density in the left parietal lobe. The most likely diagnosis is:A. Multiple sclerosisB. StrokeC. AstrocytomaD. AbscessE. MetastasisView Answer22. The answer is C [XVII A 1 a]. The patient presents with a 2-month history of progressive neurologic deficits. The CT scan shows an area of low density, and there is no mention of mass effect. The most likely diagnosis is an astrocytoma, which may not have much in the way of mass effect or enhancement on CT scan.The progressive history is against a stroke. Multiple sclerosis rarely presents in this age group and would not be expected to cause a large area of decreased density on CT scan. An abscess or metastasis would have substantial mass effect and enhancement on CT scan.